Форум » Флейм » Что такое Ri? » Ответить

Что такое Ri?

RedStar: Прошу высказать полное обоснование необходимости расчетов Ri. Сколько сталкиваюсь, столько прихожу к мнению, что знать Ri нет необходимости.

Ответов - 299, стр: 1 2 3 4 5 6 7 8 9 10 11 12 13 14 15 All

Пермяк: Попробую (в очередной раз) пояснить, что такое Ri, попроще. RedStar, нагрузка, включенная в ламповый каскад, "видит" лампу, как источник переменного напряжения (или источник переменного тока), имеющий именно такое внутреннее сопротивление, как Ri данной лампы. http://hiend.borda.ru/?1-13-20-00000091-000-0-0-1427557566 RedStar, опробуйте хотя бы осмыслить, не торопитесь комментировать и выкладывать свои открытия.

Кузьмич: RedStar, лампа для нагрузки является генератором переменного напряжения. Если вместо лампы поставить генератор с теми же выходными параметрами напряжения и тока, то получишь то же самое. Пермяк пишет:Попробуйте хотя бы осмыслить,Неоднократно замечал, что он путается, скажем так, с сопротивлением лампы по постоянному току, т.е. Ua/Iа. Да ещё хочет поймать этих блох в разной точке нагрузочной. Забывая или не понимая того, что для определения Ri в рабочей точке, напряжение на сетке должно быть постоянно, конста́нта.

RedStar: Кузьмич пишет: Ri при расчётах каскадов УНЧ принимается в рабочей точке, а дальнейшее его изменение в пределах нагрузочной линии, как правило не учитывается. Если учитывается в РТ, а далее нет, почему? Потому ли, что оно будет разным и в моменты времени и при разных напряжений? Да и не про РТ идет разговор, где каждое значение будет константой относительно других. При подсчете эквивалентного сопротивления, вообще ничего не получится, так как как раз не учитывается выше приведенный фактор при разных значениях по нагрузочной. Генератор - устройство, производящее какие-либо продукты, вырабатывающее электроэнергию или преобразующее один вид энергии в другой. Ключевое слово - вырабатывающие. Но лампа преобразует постоянное напряжение в переменное под действием внешнего переменного напряжения. Пермяк пишет: НАГРУЗКА "видит" лампу, как генератор переменного напряжения (или генератор переменного тока), который имеет внутреннее сопротивление, равное Ri лампы. Нагрузка какая? Про нее потом.


Stan Marsh: Краснозвёздыч, Вы бы засунули свой гонор туда, где ему место, и честно признались бы в том, что просто не понимаете. Глядишь, и дело бы пошло. Так нет! Пытаетесь сохранить лицо. А со стороны видна рожа, без проблесков интеллекта, с тупой гордыней и без желания учиться. И лично у меня нет никакого желания Вам помогать.

RedStar: Гордыня скорее у вас всех прет от великого знания книжных истин. Что то не понимаю, спрашиваю. Только ответы всегда отрицательный у вас на мои вопросы. Верно, сохранить "свое лицо" пытаюсь, а ваши "рожи" пропитаны брюзжанием. Учиться можно и нужно. Только без Хороших учителей вряд ли получится у кого либо.

Кузьмич: RedStar пишет:Генератор - устройство, производящее какие-либо продукты, вырабатывающее электроэнергию или преобразующее один вид энергии в другой. Толя, в очередной раз удивляюсь, какой ты глупый... Это общее понятие генератора. Превращение механической или тепловой... энергии в электрическую. А ты слышал когда-нибудь про мост Ви́на? Кажется так звали немецкого физика. Который, скажем так, изобрёл электронный генератор синусоидальных колебаний. Сам мост состоит из пассивных элементов. А если его включить в цепь положительной обратной связи усилителя, выполнить условие баланса фаз и амплитуд, то усилитель превратится в генератор гармонических колебаний. RedStar пишет: Нагрузка какая? Про нее потом. Нет, дорогой. Лампа без нагрузки - это кусок стекла, металла и карболита. Ничего ты с неё кроме тепла не получишь. Поэтому - только и только с нагрузкой. Оную выбираешь сам, исходя из Ri в рабочей точке, которую сам же и определишь....Если учитывается в РТ, а далее нет, почему?Потому что, решив собрать схему на лампе, ты и только ты выбираешь для неё режим, т.е. РТ с которой она начнёт работать. И все её диф. параметры берутся или рассчитываются, или измеряются исходя из выбранной ЭТОЙ РТ. Исходя из этих параметров рассчитывается эквивалентное сопротивлений (оно нужно больше для НЧ), задаётся Альфа и т.д, и т.п. ... Всё остальное о чём ты думаешь - никакого значения не имеет. Я не зря привёл аналогию с сетевой розеткой 220 вольт... "Бьёт" всегда 220, не зависимо от того, что замкнул собой в тот момент, когда было 25 вольт, или 57, или больше. Ты задаёшь вопросы, типа: - Почему апельсины круглые? - Почему в дырках ничего нет? Не забивай голову посторонними вещами.

RedStar: Не приписывайте мне, что я не знаю как высчитывается Ri. Хочу уточнить одну деталь, почему должно увеличиться (или уменьшиться) анодное напряжение, при константе Мю, в дин. режиме, с выбранной высокоомной нагрузкой?

Плюмбум: Кто-нибудь понял, о чём спрашивает RedStar ?

Пермяк: Мне кажется, его вопрос можно перефразировать так: почему коэфф. усиления каскада (Ку) всегда меньше, чем Мю (µ) лампы? Анатолий, я угадал?

Кузьмич: Пермяк пишет:Анатолий, я угадал? А я пыаюсь понимать по другому. Ведь RedStar пишет: с выбранной высокоомной нагрузкой?Например взята лампа 6Н2П или 6Н9С, или 6Г2... Где сопротивление анодной нагрузки высокоомное и может быть 150, 200, 270 и даже более кОм. Почему при такой огромной нагрузке напряжение на аноде должно изменяться под воздействием входного сигнала (т.е. при изменении сеточного напряжения)???? Кажется так надо понимать его вопрос. Только при чём там Мю константа, не понимаю...

RedStar: Пермяк пишет:я угадал? Нет. Кузьмич пишет:Почему при такой огромной нагрузке........Нет. Почему должно изменится анодное напряжение дельта Ua при нагрузке Ra?

Кузьмич: RedStar пишет:Почему должно изменится анодное напряжение дельта Ua при нагрузке Ra? А дальше читаем и осмысливаем, потом запоминаем учебник. Например вот этот: Т.е. не почему должно измениться, а на сколько надо изменить анодное напряжение, что бы анодный ток изменился на 1 ма, в рабочей точке, которую выбрал самодельщик, т.е. ты. И при чём тут Ra?. RedStar пишет: Не приписывайте мне, что я не знаю как высчитывается Ri.Дело значительно хуже, ты не знаешь что такое Ri.

Пермяк: На рисунке Редстара изображён характеристический треугольник для раб.точки красного цвета. Поскольку гипотенуза этого треугольника - касательная к раб, точке, то такой треугольник предназначен для определения Ri в данной точке: ∆Ua/∆Ia=Ri (напряжение делённое на ток - это ведь сопротивление, не так ли?) Если рассматривается трансформаторный каскад, то при изменении нагрузки изменится угол наклона нагрузочной при той же рабочей точке А, а величина Ri не изменится: ЗЫ. Однако, о каком Мю ранее спрашивал Редстар, не понятно. ЗЗЫ. Ох, Куззмич опять опередил, экий торопыга!

RedStar: Все не поняли вопроса. Кузьмич пишет: не почему должно измениться, а на сколько надо изменить анодное напряжение, Зачем изменять напряжение? Что бы узнать Ri, это-то понятно. Но Почему должно изменится анодное напряжение в конкретной схеме (режиме)??? В РТ установлен режим "А". Каким образом изменится Ua0 при изменении нагрузки? Пермяк пишет:при изменении нагрузки изменится угол наклона нагрузочной при той же рабочей точке А, а величина Ri не изменится Само собой. Вы все так говорите, будто я не знаю.

Кузьмич: RedStar пишет:Зачем изменять напряжение? (что бы узнать Ri, это то понятно) Чудеса! Тогда не понятно, что тебе не понятно. RedStar пишет: но Почему должно изменится анодное напряжение в конкретной схеме (режиме)??? Да не должно ОНО меняться! Ты сам себе ответил на вопрос: RedStar пишет: (чтобы узнать Ri, ...)

RedStar: Кузьмич пишет:Да не должно ОНО меняться! Тогда зачем считать Ri? Если оно дает неточный расчет нагрузки.

Пермяк: RedStar пишет: Тогда зачем считать Ri? Если оно дает неточный расчет нагрузки. Ri - это параметр ЛАМПЫ. А нагрузку мы не рассчитываем, а ВЫБИРАЕМ, задаём, например для триода - в пределах (3...5...7)Ri, и более. Желаем побольше мощность - выбираем 2Ri, но Кг будет относительно большим, желаем меньшее Кг - выбираем (3...7)Ri, но мощность на выходе получим меньше. Какая тут может быть точность? Всё зависит от наших предпочтений.

RedStar: Кузьмич пишет: Что значит не точный? Такой вот не точный: Пермяк пишет: нагрузку мы ВЫБИРАЕМ, например для триода - в пределах (3...5...7)Ri, и более. Кузьмич пишет: И какая точность тебе нужна? Расчетная, до 5%. Выбирая Ra от Ri, ее не получить никогда.

Кузьмич: Можно и не считать Ri, а взять его из даташита, с условием, что РТ будет такая же, как указано в даташите. RedStar пишет: Расчетная, до 5%. Но с Ri ее не получить никогда. Получишь и даже меньше. Для этого тоже есть вариант, снимаем самостоятельно ВАХи конкретной лампы и считаем Ri в РТ. А лучше пользоваться программами. Это удобно, быстро и точно. Сайчас множество ВАХов различных ламп, снятыми людьми для облегчения. П.С. Когда года подходят к 70... остро понимаешь, как мало мы живём. И как много потеряно ценного времени на всякую хрень.

Пермяк: RedStar Повторюсь: если мы изменяем нагрузку по своей прикидке, зачем тебе точность 5% ?

RedStar: Кузьмич пишет:Получишь и даже меньше. Для этого тоже есть вариант, снимаем самостоятельно ВАХи конкретной лампы и считаем Ri в РТ. Самостоятельно можно снимать, но зачем? Есть готовые ВАХи. Как же можно меньше %? Приведите пример. Пермяк пишет:Если мы изменяем нагрузку по своей прикидке... Я не изменяю нагрузку. Ее рассчитываю сразу. П.С. Пермяк пишет: А при увеличении Ra напряжение Ua0 приходится задавать бОльшим (при этом часто - и ток Ia0 снижать) чтобы вых. мощность не снижалась, или снижалась не так сильно. Так это справедливо для выходного каскада, и то не обязательно, если потеря мощности не важна. А для входного не обязательно это условие, если не превышать мощность анода.

Пермяк: RedStar пишет: Я не изменяю нагрузку. Ее рассчитываю сразу. Не получится "сразу". Всегда приходится делать несколько заходов. И полученный результат всё равно не обеспечить с такой точностью, т.к. ещё не имеешь точного расчёта ТВЗ, который считают уже после расчёта каскада. ЗЫ. А как у тебя с расчётом ТВЗ? Неужели научился? С учётом твоих КО и ЭО ? RedStar пишет: Самостоятельно можно снимать, но зачем? Есть готовые ВАХи. Готовые? В даташите? Читаем даташит: Посчитайте процент разброса.

RedStar: Для входных, драйверных, каскадов рассчитывается сразу, без Ri. Пермяк пишет:ещё не имеешь точного расчёта ТВЗ, который считают уже после расчёта каскада.А если все вместе считается? ТВЗ вообще по другому рассчитываю. Про ЭО долго думал. Нашел принцип, похож на тот, что ранее делал ветку об ЭС, но еще не довел к основанию. П.С. Про даташит. Мне же не надо Ri, я про график ВАХ.

Пермяк: RedStar пишет: Мне же не надо Ri, я про график ВАХ. Если Ri имеет разброс, то и графики ВАХ - такой же. Ri ведь снимается именно с ВАХ. Припомните Ваше недоумение по поводу несоответствия в разных даташитах ВАХ ГУ-50 в пентоде...

RedStar: Есть график. Нужны только его ветви и оси, по которым находится нагрузка и прочее, не прибегая к вычислению и даташитного Ri. Да без этой Ri все считается. Пусть имеется разброс. А смещение для чего? Им же можно полуоси нагрузки выставить относительно РТ. Пермяк пишет:Припомните Ваше недоумение по поводу несоответствия в разных даташитах ВАХ ГУ-50 в пентоде... Было дело. Теперь это можно обойти.

Кузьмич: RedStar пишет:Как же можно меньше %? Приведите пример. Толя, не понимаю, тупишь??? Я же объяснил. Ну хорошо, снимаешь ВАХи, своей конкретной лампы Например 6С3П, при различных напряжениях анода, те, которые ты желаешь применить. И вычисляешь Ri. Буржуйские приборы обозначают это Rp. RedStar пишет: без этой Ri все считается. Это у кустарщиков всё так считается. А ты на техническом форуме. И к примеру, твоя "транзисторная", но суперская акустика (8 Ом), не может нормально воспроизводить с усилителем, вых сопротивление которого больше 1 Ом. И как ты будешь рассчитывать вых. сопр. усилителя без Ri??? Расскажи. Как кустарщик???

RedStar: Кузьмич, зачем мне Ri? А дальше расчет вычисления Ra, ...? Из десятков 6с3п и 6с4п ни одна не подходит под ВАХ Клауса. Под "наши" - идеально. РТ для них Ua=140, Uсм=-2,0. Ra=30к Кузьмич пишет:А ты на техническом форуме.Именно. Разве технически нельзя применить другое вычисление?И как ты будешь рассчитывать вых. сопр. усилителя без Ri??? Расскажи. Как кустарщик???Почему выходное сопротивление считается от разницы нагрузок? Ктр меняется, - меняется переменный ток анода. Ri остается неизменным, а Ra изменяется. (Выше, у Леонида, пост N: 6334)

Кузьмич: RedStar пишет:зачем мне Ri? Чего ты тогда пристаёшь к народу??? Вот за это высказывание его надобно и близко не подпускать к форуму.

RedStar: Кузьмич пишет:Чего ты тогда пристаёшь к народу??? Нужен принцип его применения. Зачем и для чего, расчетами. В сравнении познается разница методик. П.С. Кузьмич, не надо категорично высказываться. Нравится вам это или нет, - не дает право делать такие выводы.

r9o-11: А Вы читали описание к программе Tube TransCalc? Здесь лежит Там есть формулы, применяемые в расчётах и по ним можно посмотреть что к чему привязывается и как зависит - разве это не принцип применения?

Кузьмич: RedStar пишет:Нужен принцип его применения. Зачем и для чего Всё в учебнике, который тебе не под силу. Остальное - всё твои выдумки = чушь!

RedStar: r9o-11, спасибо. Кузьмич пишет:Остальное - всё твои выдумкиЭто как посмотреть. Всего лишь задаю вопросы и нужны ответы. Я не предлагаю вам это как истину. Сравнение для меня важно.

rv10: RedStar пишет: Ra изменяется, а Ri остается неизменным. И поэтому Ri знать не нужно?

RedStar: rv10, так объясните популярно, зачем нужно Ri. Никто не отвечает по сути, полным объемом. П.С. Есть хоть какая книга, где существует практические работы с теоретическими расчетами объяснения Ri? Сказал же - я сравниваю разные методики!

r9o-11: "Проектирование схем на компьютере" Васильченко и Наседкина читали?

Aleph: Не понимаю что тут во внутреннем сопротивлении сложного. Допустим мы даже ВАХов лампы не знаем. Просто подали напряжение на неё (при постоянном смещении) и получили определенный ток. По закону Ома мы узнаем сопротивление лампы в этой точке. При других режимах будет и другое сопротивление. Если бы сопротивление не менялось с режимом, лампа бы являлась обычным резистором. Готовые ВАХи нам нужны уже для того, чтобы выбрать такую нагрузку чтоб усиление лампы на положительной и отрицательной полуволне было максимально одинаковым. Как люди при одном размере одежды всё же отличаются, так и лампы, хоть и сделанные одинаково, немного отличаются по параметрам. Мы пользуемся в жизни стандартными размерами одежды, так и с лампами пользуемся усредненными ВАХами подразумевая что они не идеально описывают работу именно нашей лампы. Что тут может быть такого мудрёного, чтоб создавать отдельную тему и не спать ночами?

r9o-11: Может быть, вечная борьба с компромиссами не даёт спать по ночам? Ведь выбрать такую нагрузку чтоб усиление лампы на положительной и отрицательной полуволне было максимально одинаковым нужно при некоторых условиях, которые часто связаны между собой и нередко противоречат друг другу.

Пермяк: Aleph пишет: подали напряжение на неё (при постоянном смещении) и получили определенный ток. По закону Ома мы узнаем сопротивление лампы в этой точке. При этом мы получите значение Ro (Эр нулевое) - сопротивление лампы по ПОСТОЯННОМУ току (для практических расчётов ненужное) а не Ri - внутреннее сопротивление ПЕРЕМЕННОМУ току. forensic anthropology working conditions

RedStar: Aleph, не совсем понял, что имеете в виду. Не зная ВАХ лампы, можно применить даташит, в котором указаны максимальные значения. Исходя из них, делаем поправку на уменьшение. Собираю макет, где рассчитываю своим методом и общепринятым методом. Знаете, общепринятым, получается полная "лажа". Подсчитайте по всему диапазону от 2 до 10 Ri (для триодов, и 0,1-0,5 для пентодов). Что получится перебирая такое количество неизвестных? r9o-11 пишет:выбрать такую нагрузку чтоб усиление лампы на положительной и отрицательной полуволне было максимально одинаковымДело скорее не в нагрузке, ее влияние только на 60% примерно. Больше влияет напряжение смещения, которое как раз определяет положение РТ для равнозначности полуволн. Нагрузка дает оптимум переменного напряжения и переменного тока. Отсюда, - гармонические искажения. А есть ли где расчет влияния смещения к изменению других величин, как то напряжения и Тока с применением конкретных формул? Пермяк пишет:сопротивление лампы по ПОСТОЯННОМУ току (для практических расчётов ненужное) Как это не нужное? Привел же доказательство, как с его помощью можно точно рассчитать питание и ожидаемый коэффициент усиления.

Пермяк: RedStar пишет:Как это не нужное? А так. Ua0 и Ia0 - задаём, и пользуемся их величинами, а зачем нам делить Ua0/Ia0, чтобы получить Ro? Куда потом это Ro приткнуть, где использовать?Привел же доказательство, как с его помощью можно точно рассчитать питание и ожидаемый коэффициент усиления. Прошу прощения, но НИЧЕГО ВЫ НЕ ПРИВЕЛИ.

r9o-11: RedStar пишет: Нагрузка дает оптимум переменного напряжения и переменного тока. Что такое "оптимум"? Что важнее - КПД усилителя, его КНИ и ИМД, Кдемпфирования нагрузки, рабочая полоса или какой-либо другой параметр?

RedStar: Пермяк пишет:Прошу прощения, но НИЧЕГО ВЫ НЕ ПРИВЕЛИ. При всем уважении, было мной приведено в удаленной ветке "Беседы с ...". r9o-11 пишет:Что такое "оптимум"?Это наиболее благоприятный режим исходя из выбранной РТ. Который рассчитан верно, а не длительным подбором. Если режим некорректен, то пессиум покажет отклонение от режима.Что важнее -... Когда настроено правильно и отобраны исключительные параметры, то и КПД будет высоким. Как то выше написал, что нужна совокупность расчета. Она определяет сразу два каскада - выходной лампы и трансформатора.

r9o-11: Пермяк, так, наверное, ответа и нет - это же всё компромиссы, переходящие во вкусовщину...

Aleph: r9o-11 пишет:Ведь выбрать такую нагрузку чтоб усиление лампы на положительной и отрицательной полуволне было максимально одинаковым нужно при некоторых условиях, которые часто связаны между собой и нередко противоречат друг другу. А ещё полное сопротивление АС прыгает с частотой в несколько раз и вся это идеально выверенная нагрузка оборачивается пшиком. Поэтому я перешёл на двухтакты. Там как ни прыгает нагрузка всё равно суммарно выходит симметрично при подобранных лампах. to Пермяк (на пост 6349) да, Леонид, тут я напутал с сопротивлениями...

r9o-11: Так я и про это тоже говорю - сможет ли усилитель работать с разными нагрузками или максимально хорошо только с определёнными?..

RedStar: Aleph пишет:А ещё полное сопротивление АС прыгает с частотой в несколько раз и вся это идеально выверенная нагрузка оборачивается пшиком. Абсолютно согласен. Нет такой методики, в которой это учитывается. Я только начинаю это учитывать, во избежание казусов от нагрузки. r9o-11 пишет:наверное, ответа и нет - это же всё компромиссы, переходящие во вкусовщину... Кому как. Я не вижу практического взаимодействия между Ri и Ra.

Пермяк: RedStar пишет: было мной приведено в удаленной ветке "Беседы ..." сЕщё раз повторю: ничего Вы там не привели, одни бессвязные и лишённые смысла выкладки. Без пояснений, что делаете и для чего.

RedStar: Ea = (Ro + Ra) x Io Крутизна будет от требуемой выходной амплитуды напряжения и тока. Из формулы видно как что на что влияет. Мне все заново писАть? Увольте.

Пермяк: Ea = (Ro + Ra) x IoНу галиматья же, в чистом виде, прости Господи...

RedStar: Пермяк пишет: Ну галиматья же, в чистом виде, прости Господи... Обоснуйте доказательством противоположного. Вам и всем приводил расчетом по векторной математике. Жду. rv10, от вас требую вашего обстоятельного объяснения с согласием на пост Aleph-а Вы читали мой ответ, прежде чем его повторить?

Пермяк: RedStar пишет: Обоснуйте доказательством противоположного. Вам приводил расчетом по векторной математике. Обосновывать должны Вы, поскольку пытаетесь доказать что-то своё. Причём с пояснениями, зачем делаете то-то и то-то. А мне обосновывать незачем, всё, что нужно было, я обосновал Вам 4 года назад, когда Вы были под другим никнэймом.

RedStar: Тот Ник заблокирован. Ваш форум начал читать еще в 2005 году. Мне вновь привести?: KP=PL KP=AC на оси "х" PL=CB на оси "х" KP=EF на оси "у" PL=FD на оси "у" Точка P делит отрезок KL пополам. возьмем из первого графика: H = (P + KL) x F По второму рисунку выводится так: x'/y'=z' x/y=z e = (x'/y')+(x/y)*y = (z'+z)*y или: Ea = (Ro + Ra) x Io Или скажете, что здесь математика не верна? Здесь отрицаются какие либо законы? П.С. Теперь жду вашего обоснования противоположного ответа с применением Ri. П.С.2. Или Декарт не прав?

Пермяк: Вы опять за старое? Где пояснения? Каждая, подчёркиваю: КАЖДАЯ строка с матем.зависимостями, приведёнными Вами, должна сопровождаться пояснениями: для того-то и того-то берём такую-то формулу, взятую оттуда-то. Что, Вас не учили этому? А прежде всего надо пояснить коллективу, что за дисциплина "Векторная алгеба", и для чего предназначена. Так что, Вам надо писАть статью, на форуме всё это изложить. Да, трудно...

RedStar: Хорошо. Вот пояснение. По ординате: По абсциссе:

RedStar: Пермяк пишет: прежде всего надо пояснить коллективу, что за дисциплина "Векторная алгеба", и для чего предназначена.http://kvm.gubkin.ru/vector.pdf Вам надо писАть статью, на форуме всё это изложить Согласен с вами, надо написать. Без плотно ухоженного состояния текста, вряд ли поймут. Только не могу найти того, кто поможет грамотно составить изложение.

Пермяк: rv10 пишет: нет никакой разницы между переменым и постоянным сигналом при усилении его лампой Uо и Iо - не сигнал, который требуется усилить, а установленный исходный режим ДО ПОЯВЛЕНИЯ управляющего сигнала, рабочий режим. У Вас плохо с пониманием даташитов? Я ведь именно из даташита выложил скан (пост 6349).

rv10: RedStar пишет:Постоянный ток не имеет частоту. Вы о чем вообще конечно не имеет но её имеет меандр который и состоит из отрезков постоянного тока. представьте периодичность импульсов час, представили полка импульса длительностью час.Только не могу найти того, кто поможет грамотно составить изло что и требовалось доказать. Пермяк, я понимаю о чем Вы, но я рассматриваю лампу и как усилитель постоянного тока, а это есть не что иное как установленный режим, или я ошибаюсь?

RedStar: Послушайте, rv10, если многого до вас не доходит, зачем пытаетесь казаться многозадачным без практических работ? У меня все основывается на практике. Попробуйте опровергнуть!? У вас есть практические работы? Сейчас физики-математики не в состоянии переварить такие данные в отсутствии знаний в этой области. И не только в этой, а и в общем. Какое обучение сейчас, такие и люди. rv10 пишет:имеет меандр который и состоит из отрезков постоянного тока. А где вы видели музыкальный сигнал (гармонический) с такой периодичностью? Это больше к вычислительной технике подходит, где Ri требуется. Ха.

rv10: RedStar Примените свою теорию для УПТ (усилитель постоянного тока).

RedStar: rv10 пишет: Примените свою теорию для УПТ (усилитель постоянного тока). Надо подумать. И вы хотите сказать, что я не знаю Ri? Прежде, чем писать подобное, определитесь, что имею в виду. Постоянные напряжения или переменные.

Пермяк: rv10, ошибаетесь. Посудите сами: если входной сигнал неизменный, тогда зачем его усиливать? Другое дело - сигнал постоянного напряжения, который может изменяться, но - мееедленно... Тогда рабочая, исходная точка всегда одна, а медленно изменяющийся сигнал изменяется относительно этой точки, которая неизменна, это точка отсчёта, опорная точка, и в каскаде УПТ даже жёстко застабилизирована.

rv10: Пермяк пишет: rv10, ошибаетесь. Посудите сами: если входной сигнал неизменный, тогда зачем его усиливать? Другое дело - сигнал постоянного напряжения, который может изменяться, но - мееедленно. Вы уклоняетесь от признания факта что усилитель постоянного тока или напряжения то есть УПТ, это усилитель именно постоянного тока или напряжения. допустим на входе имеем 10 вольт или 10 миллиампер, а на выходе имеем 100 вольт или 100 миллиампер. Всё. Никаких медленно изменяющихся сигналов нет. Плюмбум пишет:Ну,дела! Оказывается,Равен не знает, что такое рабочая точка? Не знает, что для усиления обеих волн переменного тока лампа должна быть наполовину открыта? Ошибаетесь, я знаю это, это вещь очевидная. Но разговор не об этом а о усилении постоянного напряжения, то есть неизменного сигнала: на входе 1 вольт на выходе 10, на входе 2 вольта на выходе соответственно 20. Однотактный усилитель постоянного тока:click here, click here

RedStar: rv10 пишет: Любой сколь угодно медленно меняющийся сигнал есть ток переменный.По сути, верно. Только надо смотреть, что является переменным, а что постоянным сигналом на входе. Однако, вы все, несомненно, понимаете о чем пытаюсь толковать. Только математика расставит все по местам. Так что придется искать помощь в написании статьи.

Ученик: RedStar пишет:придется искать помощь в написании статьи.Странное желание - писать статьи о вещах, которые не понимаешь.

Кузьмич: RedStar пишет:Однако, вы все, несомненно, понимаете о чем пытаюсь толковать. Ну да, мы все тут экстрасенсы...в написании статьи. Пепец! С ума сойти!

RedStar: Кузьмич пишет:Пепец! С ума сойти!Кто вам запрещает, сходите себе тихо. Кто приведет доказательное опровержение моего расчета или его несостоятельность?

RedStar: RedStar пишет:Кто приведет доказательное опровержение моего расчета или его несостоятельность? Это часть для расчета драйверного (входного) каскада. И это, отчасти, подходит для выходного каскада. Только есть дополнения объединяющее с расчетом трансформатора, которые не буду приводить. Причина проста, - ваше нежелание просто выслушать. У вас же всё переходит на оскорбление и на книжный указ. Сомневаюсь в ответе от "светил" форумчан. Толком никто здесь не написал: что же такое Ri? И почему нет точной формулы Ra из Ri?

Ученик: RedStar пишет: Кто приведет доказательное опровержение моего расчета или его несостоятельность? Предлагаешь опровергать это? RedStar пишет: И почему нет точной формулы Ra из Ri? А ещё глупее вопрос сможешь придумать? Чтобы определить хотя бы, где твой край непонимания.

Кузьмич: RedStar пишет:Застрелись, если не по твоему думаю (не научно). Да по мне - думай как хочешь, здесь чушь не пиши...И почему нет точной формулы Ra из Ri? Формула есть. Нет точной - да. Ну а ты сам можешь подумать почему? Ведь в учебнике всё расписано. Да и тебе неоднократно об этом говорили. Хорошо, скажу ещё раз. Знаю наперёд, что придумаешь очередную чушь... Потому, что мы сами выбираем, исходя из того, что нам нужно. Для триодов: Нужна бОльшая мощность и плевать на искажения Ra опт. = 2Ri Чёткая, конкретная формула! (8.26) Дедушка Цыкин, стр. 456. Но при этом триод имеет значительные искажения. И если они не устраивают, (а в SE УНЧ так и есть), то увеличивают коэффициент нагрузки, до получения приемлемой мощности и приемлемых искажений. Для пентодов такой вариант не подходит. И Ra = (0,1 - 0,5)Ri для пентодов писАлось в научно - популярной литературе, для пионЭров, т.е для начинающих. В учебниках ничего подобного не найдёшь. Там по другому...

Raven network: Вот про Ri доступно (в динамическом режиме при переменных составляющих) и понятно,и соответствует тематике и названию ветки, а то развели демагогию. RedStar, тебе - в первую очередь прочитать.

RedStar: Подсчитаем по ВАХ: http://hiend.borda.ru/?1-1-1571722521361-00000561-000-0-0#002 Какое внутреннее при смещении от -1,5 до -2,0? Какая будет нагрузка по "Чёткой, конкретной формуле"? При условии 330 вольт питания. П.С. Неужели нельзя понять, что я сравниваю. Считаю, собираю, налаживаю и проверяю совместимость. Это некорректно и не научно по вашему?

Кузьмич: RedStar пишет:Это некорректно и не научно по вашему?Некорректно выставлять сюда вот такую чушь: , и потом артачиться на замечания.Какая будет нагрузка по "Чёткой, конкретной формуле"?Опять ты за своё. Нет такой чёткой и конкретной формулы. И не будет. Существующая формула предоставляет выбор разработчику в зависимости от желаемого им усиления и искажений.

RedStar: Чушь, да. Коряво написАл. Так поправьте, а не ссыпьте оскорблениями. Сложно?

Кузьмич: RedStar пишет:Сложно?Не то слово. Потому что нормальному человеку невозможно понять, что ты там изобразил. Нормальный человек по праву считает ЭТО чушью. И это не оскорбление. Чушь - она и в Африке чушь.

RedStar: Кузьмич, это ваша фраза была про "Четкое...". Кузьмич пишет:Существующая формула предоставляет выбор разработчику в зависимости от желаемого усиления и искажений.Этот выбор - долгий поиск. Так прошу помочь, как вы считаете. От и до. Сопоставлю данные. Макет готов для эксперимента.

Aleph: RedStar, а покажите нам с нуля как Вы рассчитываете каскад с резистивной нагрузкой. Какие величины Вы используете. От чего отталкиваетесь. Возьмём для примера 6ж5п в пентодном включении. Неужели для этого нужна векторная алгебра? Мне допустим не важно что там написано в учебнике про нагрузку, я её выбираю с линейкой по ВАХам исходя из моего питания. А величина резистора сама по закону Ома выходит

Кузьмич: RedStar пишет:Этот выбор - долгий поиск. Так прошу помочь, как вы считаете. От и до. Не долгий. И он многократно короче, чем заниматься векторной алгеброй. Вот тут всё расписано доступным русским языком. Читай и запоминай.

RedStar: Aleph пишет:покажите нам с нуля как Вы рассчитываете каскад с резистивной нагрузкой. Охотно расскажу, если интересно вам, но позже. Хотелось бы увидеть сначала расчет от Ri. П.С. Глядя на ВАХ 6ж5п, с нее надо попробовать выжать 100 вольт выходной амплитуды. Было бы не плохо.

Кузьмич: RedStar пишет:Хотелось бы увидеть сначала расчет от Ri. Нет такого расчёта от Ri. В природе не существует. Есть расчёт УН, УМ (реостатные, трансформаторные, дроссельные...), в которых обязательно фигурирует Ri. Смотрим Войшвилло 1963 год "Усилители низкой частоты на электронных лампах" Например со стр. 453 и далее...

RedStar: Кузьмич пишет:Нет такого расчёта от Ri. В природе не существует.Так зачем же упираетесь, что из него получаете Ra? В одном могу согласиться. Оно хорошо, когда выбираешь Статические параметры, но не динамические с переменными величинами. Знаете, различные "допущения" касательно расчетов, приведенных в книгах, считаю не допустимыми. И, как написано у Войшвилло: "...зависят от координат начальной точки". Это много значит! (штудирую этот параграф от и до). П.С. Обещал изложить Алефу мой расчет, время идет, а ответа по Обязательному применению Ri с обоснованием так и не увидел. Что же, пока подожду. Но обязательно покажу, если позволят.

Кузьмич: RedStar пишет:Так зачем же упираетесь, что из него получаете Ra? А из чего же??? Задаёмся коэффициентом нагрузки А (Альфа) и считаем Ra. Ra = A * Ri (предварительно должны выбрать РТ на ВАХ и определить Ri). На основании полученного Ra, на ВАХах рисуем нагрузочную (динамическую) через РТ, по которой вычисляем мощность и нелинейные искажения. Это главное, от чего идёт расчёт УНЧ. Не от Ri, а от желаемой мощности и допустимых нелинейных искажений. А ещё... КПД. И если полученные данные удовлетворяют заданным (задуманным), то оставляем... Если нет, меняем рабочую точку, уточняем Ri, и или меняем Альфу, уточняем Ra, рисуем, считаем... Или выбираем другую лампу и всё заново. Это так называемый классический расчёт. В радиолюбительстве бывает несколько иначе. Есть лампа, её хочу... Сколько даст мощности, столько даст. А нелинейных чаще хотят получить поменьше. Всё выше сказанное - применительно к триодам. При определении оптимального Ra для пентодов Ri обычно не учитывают. Там несколько иначе. RedStar пишет:как написано у Войшвилло: "...зависят от координат начальной точки". Это много значит! (штудирую этот параграф от и до) ... Правильно. От координат РТ (U0 и I0) много чего зависит. И их надо выбрать правильно. Подозреваю, ты собрался сюда впихнуть векторную алгебру: "допущения" касательно расчетов, приведенных в книгах, считаю недопустимыми. Это от твоего низкого образовательного уровня. Ты не понимаешь сути допущений.а ответа по Обязательному применению Ri с обоснованием так и не увидел. Ri необходимо для расчёта некоторых параметров усилителя (каскада), на стадии его проектирования. Самодельщикам-кустарщикам этот параметр зачастую не нужен. Да они про него просто и не знают. Как и ты.

Пермяк: Кузьмич, судя по всему, Редстар в этой теме пишет о резистивном каскаде. А мы ему рассказываем про трансформаторный...

r9o-11: Так разве основной задачей резистивного каскада является передача мощности в следующий каскад? Это же совсем не схемотехника транзисторных ВЧ усилителей мощности с их "омными" входными сопротивлениями... Зачем в предварительных каскадах альфу считать?

Кузьмич: Пермяк пишет:судя по всему, Редстар в этой теме пишет о резистивном каскаде. Принцип определения нагрузки триода в резистивном каскаде, мало чем отличается. И сводится к достижению максимального усиления сигнала и допустимых частотных искажений на ВЧ. При вычислении Ra~ необходимо учитывать Rc следующего каскада, т.к. Ra и Rc соединены параллельно. Учебники "рекомендуют" нам выбирать Ra для триода не более (5 - 10)Ri. П.С. Краснозвёзному, по незнанию, не нравится такое допущение. Ему надо, что бы было строго, например 3,75Ri... Кажется так. Расчёт нагрузки для пентодов производится несколько иначе...

RedStar: Кузьмич пишет:При вычислении Ra~ необходимо учитывать Rc следующего каскада, т.к. Ra и Rc соединены параллельно. Само собой. Всегда при макетировании ставлю Rc следующего каскада. Только вот неувязочка получается. Если они параллельно, то суммарное сопротивление должно быть много меньше. А этого нет. Ra*Rc / Ra+Rc - верно? Увы, не получается так.Ему надо, чтобы было строго, например 3,75Ri... Да, так и надо. Может быть 2,857 или 3,2144? Когда точнее рассчитываю, меньше Кг%. Для пентодов, верно, иначе. Но суть то остается. Плохого учебники не пишут. Но все они ориентированы временем становления ламповой техники, где питание было кенотронное. Все последующие, - лишь копия старых книг с незначительными дополнениями.

Ученик: RedStar пишет: Если они параллельно, то суммарное сопротивление должно быть много меньше. Какое такое "суммарное" сопротивление? Почему "много меньше"? У тебя Ra и Rc одного порядка, что ли?Ra*Rc / Ra+Rc А что ты так считаешь - то?Да, так и надо. Не занимайся ерундой. Никогда Ra не выбиралась и не будет выбираться в каком-то строгом соотношении к Ri. Почему - тебе писали много раз, но ты же не читатель... Когда точнее рассчитываю, меньше Кг% Чего и как ты рассчитываешь - тайна за семью печатями. Оформить свои изыскания в приемлемую форму ты не в состоянии. P.S. Какая может быть векторная алгебра? У тебя с арифметикой нелады.

Кузьмич: Ученик пишет: Какое такое "суммарное" сопротивление? Это смоленский диалект, понимать надо... RedStarменьше Кг% Тогда бери 10 - 15, 20 Ri. Или ещё больше, коль только Кг% интересует. Но не плачь, когда столкнёшься с другими трудностями.

RedStar: Кузьмич пишет:Тогда бери 10 - 15, 20 Ri. Зачем? Там питание будет велико. Тогда из каких критериев исходить при расчете? От имеющегося питания или, все же от РТ, где потом питание делать?

Кузьмич: RedStar пишет:Тогда из каких критериев исходить при расчете? Ну я же тебе всё дал в ссылках. Читай, запоминай, выписывай... По поводу искажений. Во многих случаях примерно до 5% искажений ты можешь даже и не заметить, что они там есть. Зависит от спектра искажений.

aleks8845: Кузьмич пишет:Зависит от спектра искажений. Тут - в точку, спектр очень важен. Малейшее превышение 3й гармоники относительно 2й или наравне, иль вообще больше 2й - звук начинает «подвизгивать»...

RedStar: Кузьмич пишет:Ну я же тебе всё дал в ссылках. Это для трансформаторного. Хотя для резистивного тоже подходит 2...10*Ri. А как быть, если ограничено питание некой величиной и из нее надо строить нагрузочную? Перебирать 2...10? Как-то напряжённо. А в случае с трансформатором, - перематывать многократно? Допустим, зная питание и выбрав на графике приемлемую РТ для резистивного каскада, как же можно точнее рассчитать со статическим данными на лампу? Причем отдельно, триод и пентод.

Aleph: RedStar пишет:Да, так и надо. Может быть 2,857 или 3,2144? Когда точнее рассчитываю, меньше Кг%. Для пентодов, верно, иначе. Но суть остаётся. Толку в этих точных расчётах, если ВАХи - усредненные, и очень часто снятые собственноручно выглядят покривее справочных. Даже если снимете ВАХ для своей лампы, в течение службы она меняет свои параметры. Кроме того, параметры плавают вместе с напряжением смещенеия (или Вы все питания стабилизируете с точностью до трех знаков? А что со сменой отслужившей лампы? Под нее потом новый анодный резистор подбираете, тоже с точностью до 3 знаков? Читал недавно, что есть такое заболевание, когда даже легкая непараллельность столовых приборов, или скажем на сантиметр короче галстук, выводят из себя до нервного срыва. Кажется, любовь к излишне точным цифрам тоже туда относится

RedStar: Aleph пишет: Толку в этих точных расчетах если ВАХи усредненныеНе скажите. Даже если ВАХ усредненнные значительно, то совсем небольшой подстройкой автоматическим смещением или фиксированным, всегда можно "выправить" нелинейность.анодный резистор подбираете, тоже с точностью до 3 знаков? Зачем же? По расчетным собираю иногда сборку. К примеру, надо 6245, ставлю 6200. Или надо 15300, - ставлю подбором тестером ближайшие к этому сопротивлению.Кажется, любовь к излишне точным цифрам тоже туда относится Эти цифры не лишние. Как правило - сразу удается получить требуемое.

Пермяк: RedStar Хм, ратуете за точность? Какими бы точными ни были ВАХи, Вы ведь в своих схемах радикально их изменяете: - в резистивном драйвере - применением питания экр. сетки с части анодной нагрузки (как бы УЛ); - в выходном каскаде - применением КО и ЭО. И ничего заранее вычислить невозможно. Даже с помощью векторной алгебры. Только утомительным подбором :)

RedStar: Пермяк пишет: ратуете за точность? ... ничего заранее вычислить невозможно. Верно, ратую. И вполне с этим справляюсь. Все же можно. Что в резистивном, что в трансформаторном. А эта часть векторной алгебры как раз нужна была для основы. Совсем близко добрался до вычисления ЭО. Немного додумать и сопоставить, только по времени долго придется работать с макетированием.

Пермяк: RedStar пишет: эта часть векторной алгебры как раз нужна была для основы. Мало пояснена эта часть (кстати, которая?), надо подробнее пояснять, я Вам уже советовал...

RedStar: Которая расписана, и довольно подробно: http://hiend.borda.ru/?1-22-1572087598365-00000544-000-80-0#041 Больше добавить нечего, и нет желания из-за неадекватности некоторых форумчан, так как оскорбления принимаю всерьез. Больше рассказывать о своем методе не буду. Варитесь дальше в прошлом.

Ученик: RedStar пишет: Которая расписана, и довольно подробно А что там является целью вычислений? Напряжение анодного источника и сопротивление нагрузки?

RedStar: Абстрагирование от Ri. Какая эквивалентная схема является последовательной (Ri+Ra), а какая параллельной? Резистивная или трансформаторная?

Ученик: RedStar пишет: Абстрагирование от Ri. Хм...оно и при обычном расчёте не является обязательной отправной точкой. Уж для резистивного - точно. Да и для трансформаторного его учёт не всегда обязателен. Я вот вижу, что в начале расчёта тебе известны точки K, P и L. Откуда ты взял, или предполагаешь брать эти данные? Какая эквивалентная схема является последовательной (Ri+Ra), а какая параллельной? Резистивная или трансформаторная? Смотря что тебя интересует. Если выходное сопротивление каскада - параллельная. А если, например, влияние пульсаций анодного источника - последовательная.

RedStar: Так зачем же "отправной точкой" используете Ri для вычислений, что резистивных, что тран-ных? Включая Альфу и примерные значения от 2 до 10 (или 0,1-0,5) Ri. Ученик пишет: в начале расчёта тебе известны точки K, P и L. Откуда ты взял, или предполагаешь брать эти данные? Известна только Рабочая точка, которую выбираем. Остальное зависит от требуемой выходной амплитуды.Если выходное сопротивление каскада - параллельная. А если, например, влияние пульсаций анодного источника - последовательная. Это как? В транс-ном - последовательная? Нет?

Кузьмич: RedStar пишет: (или 0,1-0,5) Ri А это что? И откуда взято???Это как? В транс-ном - последовательная? Нет? По переменному току верхний "конец" по схеме анодного резистора, или верхний "конец" обмотки трансформатора "сидит" на общем проводе через конденсатор большой ёмкости БП, т.е. параллельно лампе, т.е. её внутреннему сопротивлению Ri. А этот конденсатор большой ёмкости БП на НЧ( не говоря уже для более высоких частот) имеет сопротивление близкое к нулю.

RedStar: Кузьмич пишет:Дожили...Покажите пример. (не ради забавы). То, что: Кузьмич пишет: По переменному току верхний "конец" по схеме анодного резистора, или верхний "конец" обмотки трансформатора "сидит" на общем проводе через конденсатор большой ёмкости БП, Естественно. А между транс-ром (нижний вывод тр-ра) и лампой (анодом) есть что либо? Считаете это параллельным? Добавлю: "...ток пойдет по цепи из последовательно соединенного га с параллельной ветвью RL и LP ." http://www.vestnikara.spb.ru/vestn/n3/moir.htm

Кузьмич: RedStar пишет:Покажите пример. (не ради забавы). Цыкин, раздел 5,3 Трансформаторный каскад. Стр. 132 и далее.А между транс-ром (нижний вывод тр-ра) и лампой (анодом) есть что либо? Есть! Твоя любимая амплитуда, которую надо втиснуть куда-то далее. Ещё раз спрашиваю: Откуда это взято: RedStar пишет: (или 0,1-0,5) Ri.???

Ученик: RedStar пишет: Так зачем же "отправной точкой" используете Ri для вычислений Кто использует????Известна только Рабочая точка, которую выбираем. Остальное зависит от требуемой выходной амплитуды. Рабочая точка - это одна точка. У тебя - три. Объясни, откуда ты берёшь точки K и L.Это как? В транс-ном - последовательная? Хоть в каком. Выше написал: если рассматриваешь влияние пульсаций анодного источника, по простому - фон от этих пульсаций, то здесь рассматривается как обычный делитель: Rа включена с Ri последовательно.

Пермяк: В начале этой темы RedStar писал:До сих пор не пойму реальное применение, хотя прекрасно знаю, что представляет из себя внутреннее.зачем нужно Ri. Никто не отвечает по сути, полным объемом.1. Ri драйверной лампы нужно знать, чтобы вычислить выходное сопротивление драйвера Rвых. Если каскад резистивный, то его выходное сопротивление равно: Rвых=Ra||Ri Если Rвых составляет 10% и более от величины гридлика выходной лампы, то разделительный конденсатор Ср рассчитывают с учётом Rвых драйвера. При наличии в драйверном каскаде ООС (например, Rк не зашунтирован ёмкостью), то Ri.св лампы возрастёт: , и его тоже надо узнать (вычислить), чтобы определить величину Ср. Некто может возразить, что Ср легче подобрать, чем рассчитывать, но, во-первых, для этого нужен запас разных по ёмкости конденсаторов, а во-вторых, зачем тогда нужна векторная алгебра, если детали подбираем "методом тыка"? 2. Ri лампы в выходном каскаде также участвует в формировании выходного сопротивления: Чем меньше Rвых, тем лучше коэффициент демпфирования подвижной системы динамика. Вывод: Ri - важный параметр лампы, и его, так же, как как µ и S желательно знать.

Aleph: RedStar пишет:Так зачем же такой "отправной точкой" используете Ri для вычислений, что резистивных, что трансформаторных? Включая Альфу и примерные значения от 2 до 10 (или 0,1-0,5)Ri. Да никто из умеющих проводить расчёт не использует эти соотношения, они просто показывают, в каких примерно пределах может находиться значение Ri, для новичков, просто для сведения. Кто умеет - строит нагрузку по ВАХ.

RedStar: Ученик пишет: Объясни, откуда ты берёшь точки K и L.Зачем? Вы же все считаете, что у меня всё идет вразрез физике и закону дедушки Ома. Кроме РТ есть еще два значения, - питание - Ea, и требуемая выходная амплитуда. От них надо исходить. Aleph пишет: никто из умеющих проводить расчёт не использует эти соотношения, они просто показывают,... Далее и без Ri можно обойтись, верно? Уже говорил, что даже по примерным ВАХ можно точно настроить каскад. Ведь только формулами можно доказать выходное сопротивление. Или нет?

aleks8845: RedStar пишет: Далее и без Ri можно обойтись, верно? Если можно так выразитmся, пусть меня поправят, при определении РТ пусть Ri не будет являться? самым главным расчетным параметром , но в дальнейшем ,оно (Ri), все равно, будет иметь значение. В резистивном каскаде будет влиять на выходное сопротивление каскада, а в трансформаторном.... К примеру мне нужен трансформаторный драйвер, я буду выбирать к МКТ низкоомную лампу, т.е. с бОлее низким Ri, и уж точно не поставлю ничего в пентодно, тетродном включении, где Ri бывает до сотни кОм...

Пермяк: RedStar пишет: и без Ri можно обойтись, верно? Моим постом выше я показал, для чего нужно ЗНАТЬ величину Ri. RedStar пишет: Ведь только формулами можно доказать выходное сопротивление. Или нет? Представьте , что нагрузка немного изменилась. например, Rдинамика уменьшилось на НЧ. Уменьшится и напряжение на динамике. И изменится оно тем сильнее, чем выше значение Rвых, которое зависит от Ri выходной лампы. Формулу я показал, поэтому вот эта Ваша Фраза свидетельствует, что Вы не не поняли:Ведь только формулами можно доказать выходное сопротивление. Или нет? Повторюсь: падение напряжения при снижении Rнагрузки показывает, что Ri - реальная физическая величина.

Ученик: RedStar пишет: Зачем? Затем. Ты ведёшь расчёт от этих точек. Так объясни, откуда ты их берёшь. RedStar пишет: Кроме РТ есть еще два значения, - питание - Ea Ты не выдумывай на ходу, Еа ты получаешь далее из своего расчёта. А расчёт ведёшь от трёх точек. Про точку Р понятно, откуда взялись K и L?

RedStar: А для случая с пентодом, у которого 30 кОм без МОС? Какое выходное сопротивление? Около 60-100 Ом? Почему должно измениться Ri при изменении нагрузки? Изменяется ведь Ra при неизменном Ктр. Что участвует как Ra? (пункт 2 из поста Леонида). W1 = Ra? Тогда должно быть последовательно. Ученик пишет:откуда взялись K и L? Из графика, циркулем от РТ.

Ученик: RedStar пишет: Из графика, циркулем от РТ.1 Из какого "графика"? Дежурств по кухне? 2 Циркулем - это будет окружность. 3 Почему ты поставил точки в этих местах окружности - непонятно. P.S. Ты сочинил какой-то расчёт. Что является целью расчёта, откуда и как берёшь исходные данные, почему считаешь, что так будет лучше (в сравнении с чем?)...никакой информации. Почему из тебя всё клещами надо вытаскивать?

RedStar: 1. А разве не понятно, что ВАХ. 2. У вас циркулем только окружность? Про измерения им расстояний никак не догадались? Сказал ведь, что хватит про мой расчет. Ходите вокруг да около не понимая.

Пермяк: RedStar пишет: А для случая с пентодом, у которого 30 кОм без МОС? Какое выходное сопротивление? Около 60-100 Ом? Я Вам писал выше: Rвых=Ra||Ri, т.е. параллельному соединению внутреннего и анодного сопротивлений. Умеете считать сопро двух параллельно соединённых?

Ученик: RedStar пишет: А разве не понятно, что ВАХ. Так-так...У вас циркулем только окружность? Циркуль для того и придуман. А ты предлагаешь тыкать циркулем в экран монитора, или ВАХ в книге, а потом прикладывать его к линейке? Офигенная "точность", и удобство.Сказал ведь, что хватит про мой расчет. Нет, не хватит. В сухом остатке: тебя чем-то не устраивала точность графического метода. Поскрипев мозгами, ты взял те же самые графические ВАХ, но выбрал для отсчёта не крайние точки, а близлежащие к Рт. То, что точность от этого может только пострадать, в голову тебе не пришло. Ну ладно, взял и взял. Но потом проводишь какие-то идиотские громоздкие подсчёты...а просто считать с осей значения сложно? Вот же, на твоём графике Ua0 = 270 Iam = 13,5. Или же составить обычные пропорции, как в начальной школе учили - ума не хватило? Полный капец...векторная алгебра. Наткнулся:

RedStar: Ученик пишет: Циркуль для того и придуман. О как. Ци́ркуль — инструмент для черчения окружностей и дуг, также может быть использован для измерения расстояний, в частности, на картах. Причем здесь монитор? Есть принтер, распечатать, а потом проводить измерения.точность от этого может только пострадать Больше страдает точность от 2-10Ri, или в пентоде 0.2-0.5Ri. (заодно ответил на вопрос Кузьмича).Вот же, на твоём графике Ua0 = 270, Iam = 13,5. Маленько ошиблись: Ea=270, Imax=13.5. А точки Ua0 = 190, Iam = 1,0Наткнулся:Ой, тоже мне, нашли столетней давности, взятое из одной хорошей книги. И что?

Aleph: RedStar, Вы бы заявку на нобелевку дали, что ли (или может на премию Дарвина?). Почти 100 лет все считают как в учебнике, а тут Вы новый метод открыли. Да ещё и всех тайн методики не пишете, только обзываете всех неучами. Вы рискните на Вегалабе написать. Там быстро бан влепят за "тайные знания, недоступные простым физикам". Пермяк же написал доступно, хоть глотай и переваривай:Представьте , что нагрузка немного изменилась, например, Rдинамика уменьшилось на НЧ. Уменьшится и напряжение на динамике. И изменится оно тем сильнее, чем выше значение Rвых, которое зависит от Ri выходной лампы. Повторюсь: падение напряжения при снижении Rнагрузки показывает, что Ri - реальная физическая величина.

Ученик: RedStar пишет:Больше страдает точность от 2-10Ri, или в пентоде 0.2-0.5Ri. Вам уже объясняли, и не раз, что в большинстве случаев никто не считал и не считает нагрузку строго от Ri. Типа нагрузка д.б. = 4,15Ri. Иногда задаются альфой не меньше какой-то определённой величины. А какой конкретно - выбирает разработчик, хочет мощность побольше - одно, хочет высокий коэффициент демпфирования - другое. Для пентодов в звуковых цепях Ri вообще не учитывается. Сколько раз повторять?Маленько ошиблись: Ea=270, Imax=13.5. Видишь, ты понял о чём речь, ибо я не поленился, и пояснил про точки пересечения нагрузочной прямой с осями. Что же мешает тебе (уж не меньше года, наверное) сосредоточиться, и вместо обрывков-ребусов оформить свои мысли в нормальную форму? Ну так что там про точность "твоего метода"?Ой, тоже мне, нашли столетней давности, взятое из одной хорошей книги. Да из какой книги? Такая чушь только в твоей голове родиться могла. Aleph пишет: а тут Вы новый метод открыли. Да какой там новый метод. Вбил себе в голову, что нужна какая-то суперточность в этих вопросах... вот и пыжится. Сколько раз писали, что всё это усреднённое, лампы отличаются, да и нет существенных изменений от вариаций +/- 5, а часто и 10% - хоть кол на голове теши. Это называется просто - навязчивая мысль.

Пермяк: RedStar пишет: Ведь только формулами можно доказать выходное сопротивление. Или нет? Выходное сопротивление можно измерить. Причём, разными методами. Например, для трансформаторного каскада, "метод двух нагрузок": Rвых резистивного каскада можно измерить таким же методом. Нагрузкой резистивного каскада является сеточный резистор Rg последующего каскада, для драйвера - выходного. Подаём на вход каскада сигнал такой величины, чтобы переменка на Rg стала равной, скажем, U1=5В. Затем, параллельно Rg подключаем резистор R2 такой величины, чтобы напряжение на нём уменьшилось, скажем, до U2=4В. Формула для вычисления Rвых будет такой: В формуле R1 - это Rg.

RedStar: Ученик пишет: в большинстве случаев никто не считал и не считает нагрузку строго от Ri. Почему тогда строго считают Rвых от Ri? (не считая метода двух нагрузок) Пермяк пишет:Выходное сопротивление можно измерить. Спасибо за повтор. А вопрос выше.

Пермяк: RedStar пишет: Почему тогда строго считают Rвых от Ri? (не считая метода двух нагрузок) Методом двух нагрузок - ИЗМЕРЯЮТ. А с учётом Ri - ВЫЧИСЛЯЮТ. На стадии проектирования. Кстати, Ri лампы ведь тоже можно измерить. Умеете? Пробовали?

Shef: RedStar, я всегда за справедливость. Если вы несёте очевидную пургу - я вам так же об этом укажу. However, на ваших видео я увидел увлечённость кружкА энтузиастов, чего Ученик не представил. Это ахринительмо весомый аргумент, и меняет всё. ПостИте ваши видео как в дальнейшем идут дела, мы внимаемъ )) Я вам советовал заглушить комнату, в противоположной стороне от акустики, вы сделал?

RedStar: Пермяк пишет:А с учётом Ri - ВЫЧИСЛЯЮТ. На стадии проектирования.Хорошо. Вычисление выходного сопротивления от нагрузки, каким образом Ri изменяет? Вроде должны уменьшится ток и напряжение одновременно, или наоборот, увеличиться ток с напряжением. Shef пишет:Если вы несёте очевидную пургу - я вам так же об этом укажу. Я не против.Я вам советовал заглушить комнату, Комната не моя, одного из любителей. Так ему больше нравится. Только мой фотик сильно искажает при записи видео. Продолжение будет. Летний период приостановил все работы. Сейчас начинается новый виток экспериментов и прослушек. Плюмбум,там усваивать, в примере, особого труда нет. https://drive.google.com/file/d/1Ug-7P87ybPSugJS3z0e7Mvnk1me06oHB/view?usp=sharing Для Ученика: циркулем проще найти равнозначность отрезков, чем линейкой. Попробуйте на досуге. Только не на мониторе и книге, а на отдельно распечатанном листе.

RedStar: Ученик пишет: Что же мешает тебе (уж не меньше года, наверное) сосредоточиться, и вместо обрывков-ребусов оформить свои мысли в нормальную форму?Нет учителей, готовых подробно растолковать, и нет полного технического образования. Вот и вся проблема. Есть любители, нет теоретиков, в моем окружении. А практика только постепенно самостоятельности учит. И на это требуется больше времени. Вот пример, Леонид, Пермяк. Ему обязан многим. Толково пишет и в тему, без "заковыристо-оскорбительных" слов. Вам бы всем научиться его выдержке. Который раз прошу, кто же может общаться со мной вне форума? Увы, нет желающих. Только "обос...ь" можете. Жаль только то, что не все сразу понимается. Хотя есть реальные отличия от теории с практикой. Есть прямые нестыковки. Вот за них и цепляюсь. Всему нужны доказательства. Вот что интересно. Ктр трансформатора неизменно, вроде, так как не изменяется соотношение витков. Значит это не Ктр, а "n"? (как называетя?) При снижении сопротивления нагрузки или повышении, соотношение нагрузки к приведенному изменяется, - это Ктр. Индуктивное сопротивление меняется. Значит ток первички становится больше-меньше но с одновременным уменьшением-повышением напряжения. Причем тут Ri? Извините, ели опять "белиберду" написал.

Пермяк: RedStar пишет: Вычисление выходного сопротивления от нагрузки, каким образом Ri изменяет? Вычисление, как процесс, само по себе ничего не меняет. Оно просто даёт результат. Может быть, Вы хотели спросить как-то иначе?циркулем проще найти равнозначность отрезков Не равнозначность, а равенство.

XMR: Пермяк пишет:RedStar пишет: цитата: Ведь только формулами можно доказать выходное сопротивление. Или нет? Выходное сопротивление можно измерить. Причём, разными методами. Например, для трансформаторного каскада, "метод двух нагрузок": Rвых резистивного каскада можно измерить таким же методом. Нагрузкой резистивного каскада является сеточный резистор Rg последующего каскада, для драйвера - выходного. Подаём на вход каскада сигнал такой величины, чтобы переменка на Rg стала равной, скажем, U1=5В. Затем, параллельно Rg подключаем резистор R2 такой величины, чтобы напряжение на нём уменьшилось, скажем, до U2=4В. Формула для вычисления Rвых будет такой: В формуле R1 - это Rg. А этот метод подойдет для измерения выходного сопротивления пентодов и каскодов?

Пермяк: XMR пишет: А этот метод подойдет для измерения выходного сопротивления пентодов и каскодов? Подойдёт для любых каскадов.

aleks8845: RedStar пишет: Ктр трансформатора неизменно, вроде, так как не изменяется соотношение витков. При снижении сопротивления нагрузки или повышении, соотношение нагрузки к приведенному изменяется, - это Ктр. Индуктивное сопротивление меняется. Причем тут Ri? Может не все удалось понять, но к выше сказанному такие мысли пришли: Возьмем простой макет: -Лампа , в аноде резистор или Твз ( у резистора R пост. току неизменно, у тр-ра тоже ); - На выход такого каскада подключим нагрузку в виде пост. резистора ( не АС, чтоб не менялся импенданс); - Подадим Ea и переменный сигнал на управляющую сетку лампы с опр-й частотой; Если у лампы нет Ri, то на выходе каскада не будет сигнала....

Ученик: RedStar пишет: циркулем проще найти равнозначность отрезков, чем линейкой. Что значит "найти", отложить? На чём отложить, на этой линии U-I ? http://hiend.borda.ru/?1-22-1573443393804-00000544-000-60-0#039 Откуда эта линия взялась, возникла из воздуха? Это линия нагрузки, или ещё что-то? Что эти отрезки означают? Почему они равные, это по ВАХ так получилось (что сомнительно) или ты так решил? Если ты так решил - почему, есть какие-то основания, или просто почесал в затылке? Зачем откладывать два отрезка? Точки U и I, соответствующие второй точке, легко находятся простой пропорцией. Вот ты наверное думаешь, это я специально придираюсь... RedStar пишет: Нет учителей, готовых подробно растолковать, и нет полного технического образования. Так ведь даже не о технической стороне... Как-то не получается у тебя начать с самого начала, с простого изложения: вот расчёт, (чего рассчитываешь, какие цели у расчёта), данные для расчёта (откуда, каким образом взяты или получены). Вот ты пишешь: "а разве непонятно, что ВАХ". А где эти ВАХи-то на твоём рисунке? Какой лампы? Почему не на них нарисовал? - непонятно, что мешало. P.S. Упорядочить мысли очень хорошо помогает написание инструкций (считаю, что умение составить хорошую инструкцию - это искусство). Представь, что тебе надо на бумаге изложить для нормального не тупого человека инструкцию по пользованию... пусть микроволновкой. Допустим, он никогда о них не слышал, и не видел.

Кузьмич: RedStar пишет:Нет учителей, готовых подробно растолковать, и нет полного технического образования. Вот и вся проблема. Тебе здесь уже по пятнадцатомц разу растолковывают каждый пункт. Так ты всё отвергаешь и придумываешь свою какую-то хрень, типа векторной алгебры. Это равнозначно пятому колесу в телеге. RedStar пишет:XMR пишет: цитата: Мне кажется что ты не можешь доходчиво донести суть своей методики Это вернее. Какая ещё своя методика??? Методика выработана почти 100 лет назад. Всё продумано и изложена понятным, доступным языком. Если соображалка плохо работает, тупо выписать для себя что и как считать.

RedStar: Кто нарисует график изменения внутреннего Ri при переменной составляющей анод-катод и индуктивной нагрузке?

Кузьмич: RedStar пишет: Кто нарисует график изменения внутреннего Ri при переменной составляющей анод-катод и индуктивной нагрузке?Во-первых, снова написал бессвязную, несуразную чушь. Во-вторых, никто не будет, т.к. этот график никому и нигде не нужен, по причине того, что какой-либо полезной информации он не несёт. Т.е. тратить время ради твоей забавы-прихоти вряд ли кто намерен. И в-третьих, различные радиолампы имеют своё внутреннее сопротивление, изменяющееся при изменении РТ. Степень изменения так же различна. Ты собрался всё в одну зависимость определить? И в-четвёртых, если тебе это так нужно и у тебя есть много свободного времени, то проделай это самостоятельно...

RedStar: Кузьмич пишет:этот график никому и нигде не нужен, по причине того, что какой-либо полезной информации он не несёт.Если он информации не несет, нет такого графика, - зачем вам Ri? Мне тем более он не нужен.различные радиолампы имеют своё внутреннее сопротивление, изменяющееся при измерении РТ. Но если РТ неизменна? А она неизменна.

Кузьмич: RedStar пишет:Если он информации не несет, нет такого графика, - зачем вам Ri?Это твой график не несёт информации. А Ri в РТ нужна. Больше, ни в какой другой точке нагрузочной (динамической) она не нужна. Ну меняется она немного, в одну сторону в плюс, в другую в минус. Это ничего не значит, как напруга в розетке 220, от ноля до 310... - Для того, что бы определиться с Альфой и последующим расчётом нагрузки, и источника питания; - что бы рассчитать вых. сопр. каскада и Rg следующего каскада. Ты ведь разделительный конденсатор и Rg следующего каскада берёшь какой под рукой и лепишь в схему. А их, по уму, рассчитывать надо. - Да и вообще по Ri определяю, пойдёт ли мне эта лампа в каскаде или всё таки взять другую, с Ri поменьше. RedStar пишет: Мне тем более не нужен он. А нахрена тогда задаёшь вопрос: Кто нарисует график изменения внутреннего Ri при переменной составляющей анод-катод и индуктивной нагрузке? ???????

RedStar: Кузьмич пишет:А Ri в РТ нужна.Чтобы узнать, насколько изменятся ток с напряжением? С чего они там изменятся? Как верно пишете, что при разных значениях смещения - Ri разное.А нахрена тлгда задаёшь вопрос: Когда увижу абсолютную зависимость, тогда перестану задавать вопрос. Так повторю его: Кто нарисует график изменения внутреннего Ri при переменной составляющей анод-катод и индуктивной нагрузке?

RedStar: П.С. Почему же у меня, при экспериментах и сборке, не возникает вопроса расчетов через Ri и Альфу? Почему достигаю минимальные искажения всего тракта не более 0,5% при максимальном выходной мощности?Почему все напряжения соответствуют моим расчетам? Без тщательного подбора элементов и намотки трансформатора за один раз. Вот пример работы с циркулем. Где ожидается большая мощность с минимум искажений? Оба режима мною оттестированы.

Кузьмич: RedStar пишет:Что бы узнать на сколько изменится ток с напряжением? С чего они там изменятся? Нет. Тебе несколько раз объясняли для чего. Давай так: Вот тебе клиент заказал для своей дубовой резиновой акустики 8 Ом усилитель, но что бы коэффициент демпфирования был не менее 6 (шести). Иначе его акустика брыкается и плохо звучит. Про мощность ... хватит и 2-х ватт. На 10 ватт обижаться не станет. Как ты будешь выбирать выходную лампу и угождать клиенту с коэфициентом демпфирования??? Можешь рассказать??? Своими словами. Почему же у меня, при экспериментах и сборке, не возникает вопроса расчетов через Ri и Альфу? Очень просто, ты про них не знаешь. Как про суслика..., ты его не видишь, а он там есть. Плюс ещё то, что лампа, в отличие от транзистора, очень терпеливый компонент. И способна терпеть многие огрехи паяльщика. Ну, и это говорит о низком уровне знаний паяльщика.Почему достигаю минимальные искажения всего тракта не более 0,5% при максимальном выходной мощности? Вот на этом рисунке (с циркулем) не вооружённым глазом видно, что 0,5% искажений и близко нет. Гораздо больше. И если твои измерения показывают 0,5% по указанному рисунку и в указанных режимах - они далеко не точные. Да, с введением ОС искажения уменьшаются и можно получить 0,5%. Но качество звука от этого не всегда выигрывает. Только тебе это неведомо.

омельян: RedStar пишет:Оба режима Я бы выбрал: Ua0 = 210 В; Ia0 = 80 мА. Рабочая точка должна быть красивой. Тогда и искажения минимальны.

Ученик: RedStar пишет:при переменной составляющей анод-катод и индуктивной нагрузке? Переменной - какой величины? При чисто индуктивной нагрузке - зачем тебе это в звуковом усилителе? И да, Кузьмич прав, никаких 0,5% без ОС нет и не будет. Не вешай лапшу.

Кузьмич: омельян пишет: Я бы выбрал: Ua0 = 210 В; Ia0 = 80 мА.Я бы 250 вольт взял. И ток ма 70... 25 вольт смещение, ламп для драйвера огромный выбор.

Пермяк: Смотрю на рисунок RedStar'a, и - круги перед глазами... :) Для тех, кто пока не в курсе. Чтобы оценить коэфф. гармоник Кг хотя бы по 2-й гармонике, нужно иметь на ВАХ, на нагрузочной прямой, минимум три точки: - точку покоя при Ua0, Ia0, Ug0. - точку на пересечении нагрузочной прямой с кривой Ug=0, - точку на пересечении нагрузочной прямой с кривой Ug=2Ug0. (см."Красный" Цыкин, стр.75.) У RedStar'a на рисунке третьей точки нет, так что никакой оценки Кг по его графику быть не может. Анатолий, триоды и псевдотриоды имеют такое свойство: линии ВАХ неэквидистантны, т.е расстояние между кривыми на графике тем меньше, чем правее расположена кривая ВАХ. А ты откладываешь два одинаковых радиуса влево и вправо от точки покоя... Не может правая часть нагрузочной быть равной по длине левой, правая всегда короче левой. png hosting

Кузьмич: Пермяк пишет:Не может правая часть нагрузочной быть равной по длине левой, правая всегда короче левой.Отсюда и искажения. И никак не 0,5%, а поболе.: Леонид, Вам респект!!! Вот решил проверить с помощью psud2, и что получилось: Как в аптеке! С точностью до сотых. А краснозвёзному с какой точностью надо? Внутренее в рабочей точке 860 Ом.

Ученик: Кузьмич пишет: Как в аптеке! А как же иначе, если формулы одни и те же, причём правильные? Редстар как-то распинался, что Tubecurve - ужасная программа. Ну, до его "методики" конечно, не дотягивает...

Кузьмич: Ученик пишет: Редстар как-то распинался, что Tubecurve - ужасная программа. Имею основания полагать, что он не может справиться с программой.

Shef: RedStar, распечатайте на стену, рамка уже есть Кстати говоря, совсем недавно видел топик на DIYA, обсуждалась рабочая точка кажется 88 в пентоде, выбранная с заходом в область "клюшки", так, чтобы эквидистантность более-менее выполнялась.

Пермяк: Shef пишет: видел топик на DIYA, обсуждалась рабочая точка 88 в пентоде, так, чтобы эквидистантность более-менее выполнялась.Для пентода это возможно. RedStar пишет в топе: Прошу высказать полное обоснование расчетов Ri Анатолий, забыл тебе упомянуть. Есть топология с параллельной ООС, применённая О.Чернышёвым в его услителе "Покемон". В такой схеме Ri драйверной лампы входит в цепь обратной связи, и не зная Ri, невозможно провести расчёт всей схемы. И ещё, и это важно: значение Ri необходимо знать при вычмслении требуемой индуктивности первички при расчёте трансформаторного каскада: http://ulfdiysound.ucoz.ru/publ/avtorskie_stranicy/stranica_leonida_permjaka/chto_takoe_soprotivlenie_ehkvivalentnogo_generatora_pri_raschjote_l1/8-1-0-21 то есть:

RedStar: Леонид, всё, что пишете, мне известно. Еще раз спасибо за напоминание. Вот есть Ri - красным цветом, Ra - синим. Какая между ними взаимосвязь? (Почему внутреннее не может быть Ro?)

Кузьмич: RedStar пишет:Вот есть Ri - красным цветомЭто - не Ri !!! Это отрезок... Ну, можно назвать линией. Может хватит придумывать-то?Почему внутреннее не может быть Ro? А Ro - это что у тебя? Подозреваю, что это сопротивление лампы постоянному току в рабочей точке. Численно Ri может оказаться равным Ro. Только это редкий частный случай. Не всегда и не везде и не для всех ламп.Какая между ними взаимосвязь? Взамоствязь между ними - коэффициент нагрузки Альфа, который ты выбрал, задумал... (сам об этом не зная). Но это справедливо только для триода.

RedStar: Кузьмич пишет:Это - не Ri А это что у вас на графике? (такая черная линия): http://hiend.borda.ru/?1-22-1572542031509-00000544-000-180-0#080.001.001.001 Кузьмич пишет:А Ro - это что за новая выдумка? Это что??? Сопротивление постоянному току в РТ.

Кузьмич: RedStar пишет: А это что у вас на графике? ( черная линия): Это отрезок (линия) касательная к ветви ВАХ в рабочей точке.

RedStar: И разве это не одно и то же?

Кузьмич: RedStar пишет:А разве это не одно и тоже? А как может быть линия и сопротивление быть одним и тем же????? Ну сам-то подумай? Ты вроде русский человек, не папуас какой, приехавший недавно в Россию и не знающий русского языка... Это вспомогательная линия, для вычисления внутреннего сопротивления лампы. Кстати, при работе с программой эту линию не надо рисовать. Она в программе есть. Только её надо "сделать" что бы она была касательной. Тогда программа в верхнем левом углу высвечивает Ri. У басурман оно обозначается Rа

RedStar: Кузьмич пишет: Это вспомогательная линия, для вычисления внутреннего сопротивления лампы. Ну да, а я про что? Как они взаимосвязаны, Ri и Ra?

Кузьмич: RedStar пишет: Как они взаимосвязаны, Ri и Ra?Так я же написал, ты читаешь или нет? Кузьмич пишет: Взамосвязь между ними - коэффициент нагрузки Альфа, который ты выбрал, задумал... (сам об этом не зная). Но это справедливо только для триода. Ra =α*Ri

RedStar: Альфу тоже выбирать примерно надо. Не вижу связи. Значит коэффициент нагрузки можете использовать, а мне коэффициенты изменения анодных напряжения и тока от входного напряжения, - нельзя? Только не говорите, что такого не писАл.

Кузьмич: RedStar пишет:Значит коэффициент нагрузки можете использовать А почему нет? Использую всегда. Особенно при расчёте выходного каскада.а мне коэффициенты изменения анодных напряжения и тока от входного напряжения, - нельзя? А это что за такие коэффициенты??? Откуда взято???

Пермяк: RedStar пишет:Вот есть Ri - красным цветом, Ra - синим. Какая между ними взаимосвязь? Для триода - вот такая: free picture upload Хотим получить больше мощность - выбираем "альфу"≈2. Но искажения при такой нагрузке будут больше. Увеличивая "альфу" можем получить меньшие искажения, но и мощность будет пониже. Всё зависит от наших целей. Никакой другой ЗАВИСИМОСТИ нет. Этот график - теоретический. А вот - для конкретной лампы (её Ri - около 800 Ом): ЗЫ. Могу и про пентод показать зависимости, только надо ли? ЗЗЫ. И ещё, как художник - художнику: поправочка. Почувствуйте разницу.https://postimg.cc/18SXZhtN

RedStar: Пермяк пишет: Этот график - теоретический. Само собой теоретический. К практике он имеет максимум 60% достоверности. Но не об этом речь. Вот график. Сомневающийся Кузьмич не может понять, что вспомогательная линия является внутренним лампы, так как она определяет изменение напряжения и тока анода при изменении сеточного напряжения. По синим стрелкам переменное напряжение - индуктивное сопротивление нагрузки анода (без изменения частоты). Красным - показывает влияние анодного напряжения на анодный ток при постоянном напряжении управляющей сетки. Кузьмич, объясните, почему ваша "вспомогательная" линия имеет угол φ, которая составляет именно Ri? Скажите, как все же эти две величины влияют друг на друга?

Кузьмич: RedStar пишет:Само собой теоретический. К практике он имеет максимум 60% достоверности. Теоретический - значит показать как надо делать, что бы легко и просто было понять суть. Если на реальный ВАХах, практика на 60% расходится с теорией - то с практикой у этого мастера беда.Сомневающийся Кузьмич Кузьмич ни чуть не сомневается. Он полностью уверен что, RedStar пишет: что вспомогательная линия является внутренним лампы, так как она определяет изменение напряжения и тока анода при изменении сеточного напряжения. Никакая линия не является внутренним сопротивлением лампы. И линия не может определять изменение напряжения и тока. По синим стрелкам переменное напряжение - индуктивное сопротивление нагрузки анода (без изменения частоты). А это на каком языке? Или слОгане? Причём тут угол φ ? Я разве о нём что-то говорил? Хотя могу сказать, сравнивая этот угол φ на ВАХах разных ламп, можно определить у какой лампы Ri больше, а у какой меньше. Не более... Хотя опытный мастер и без угла это заметит. Пока я не пойму что ты хочешь, буду отправлять тебя учиться. Вот читай здесь про внутреннее. Да и про остальные диф. параметры полезно будет.

RedStar: Кузьмич пишет:Теоретический - значит показать как надо делать, чтобы легко и просто было понять суть. Как надо? А если НЕ как надо?Если на реальный ВАХах, практика на 60% расходится с теорией - то с практикой у этого мастера беда. Значит выкиньте эту теорию, займитесь РЕАЛЬНОЙ практикой.Никакая линия не является внутренним сопротивлением лампы. Да ладно, Вы уверены? Почитайте сами книги, в которых сказано про углы. Извинения приму, не сомневайтесь.Причём тут угол φ ? Я разве о нём что-то говорил? Вы же утверждаете: Кузьмич пишет: Это - не Ri !!! Это отрезок... Ну, можно назвать линией. У вас линия не имеющая ничего с Ri?

Ученик: RedStar пишет: вспомогательная линия является внутренним лампы, так как она определяет изменение напряжения и тока анода при изменении сеточного напряжения. Выделенное не имеет НИКАКОГО отношения к внутреннему сопротивлению усилительного прибора. почему ваша "вспомогательная" линия имеет угол φ, которая составляет именно Ri? Белиберда, что языковая, что техническая. Угол, которая составляет... 1 Прямая линия не имеет углов. 2 Две непараллельные линии в одной плоскости по определению образуют между собой какой-то угол. 3 Этот угол можно найти только после определения Ri, если оно нам заранее неизвестно. Т.е., проведя касательную (гипотенузу прямоугольного треугольника), мы легко находим катеты. Поэтому угол это бесполезен, лишняя, никому ненужная величина. Всё это учили в школе... RedStar пишет: А если НЕ как надо? Ты уж на пупе извертелся, пытаясь доказать, что "НЕ как надо" - лучше. Ничего путного выдавить из себя не можешь, оттого никого и не убедишь.Значит выкиньте эту теорию Потому что ты не способен её понять? Выкинь свой апломб... Выложенный Пермяком "теоретический" график, обрати внимание! - не для лампы конкретного типа. Это показательный график общих тенденций.Даже не знаете внутреннего при постоянке Нет такого параметра. Ri - динамическая характеристика.

RedStar: Кузьмич пишет:Вы, видимо, вскользь смотрели примерный график, раз не поняли его сути. Ученик пишет: 1 Прямая линия не имеет углов. А разве говорил, что именно "прямая линия"? Вроде на графике указано, что угол имеет гипотенузу Ri относительно Ea (по оси абсцисс). А как же угол Ra относительно оси абсцисс? Или его тоже проигнорируете?

Ученик: RedStar пишет: А разве говорил, что именно "прямая линия"? Ты свою писанину прочитать не можешь? почему ваша "вспомогательная" линия имеет угол φ, которая составляет именно Ri? Двух слов связать не можешь, ещё и споришь.Этаким "болваном" хотите меня выставить? Сам себя выставляешь. Не ищи проблемы на стороне, они - в тебе. Вот очередное подтверждение:на графике указано, что угол имеет гипотенузу Впечатление, что твоё образование закончилось на трёх классах.А как же угол Ra относительно оси абсцисс? А зачем знать величину этого угла? Провели нагрузочную линию. Получился какой-то угол между этой линией и осью абсцисс. И? Куда его приткнёшь?

Пермяк: Пишу для RedStar'а. 1. В своём сообщении № 6395 про первый график я указал, что он -"теоретический" (то есть, указывает общую тенденцию зависимости от величины "альфа"). И это несказанно обрадовало тебя, и ты написал: К практике он имеет максимум 60% достоверности.. И при этом ты сделал вид, что совсем не заметил там второй график - практический, для конкретной лампы при конкретных условиях. Нехорошо. Разумеется, Войшвилло не лично сам строил эти зависимости, а поручил кому-то из многих своих лаборантов и ассистентов. Не знаю также, этот график построен по результатам многочисленных измерений или просто получен в результате таких же многочисленных построений "на бумаге". Но то, что ты пишешь про какие-то 60% достоверности - то это очень неважно тебя характеризует. Такая погрешность ни в электронике, ни вообще в промышленности - недопустима. Твоя личная практика - не в счёт. А допустимый разброс Ri этой лампы я показал тебе в даташите: http://hiend.borda.ru/?1-22-1572670000025-00000544-000-40-0#022.001.001.002.001.001.001.001.001.002 И ещё. Ты обозначаешь угол буквой "фи". Разве ты не знаешь, что в электротехнике издревле и по сей день угол "фи" - это угол сдвига фаз между током и напряжением, а "косинус "фи" - важнейший показатель эффективности силовых (и не только) электроустройств. 2. Разумеется, Ri - это не линия, не гипотенуза, и не угол. На рис. АВ-гипотенуза, АС и ВС - катеты. Ri=ΔUa/ΔIa=АС/BC То есть, графически Ri - это отношение длины катета АС ,прилежащего к углу α (альфа), к длине противолежащего катета ВС. В математике указанное соотношение катетов называется КОТАНГЕНС угла α, т.е.: Ri=ctgα 3. Про R0. R0 не является параметрои лампы. Это всего лишь отношение напряжения и тока покоя анода, выбранных нами из своих соображений R0=Ua0/Iа0. А поэтому оно связано с работой в динамическом режиме весьма косвенно. Оно просто показывает место точки покоя на графике. И не надо его привязывать ни к внутреннему, ни к нагрузочному , вообще ни к чему. Это всего лишь начальная точка построения на графике. И не входит ни в какую формулу при расчёте.

Плюмбум: Друзья, а я разгадал великую тайну Редстара! Выбираем Ua0 и РТ. Строим мрямоугольник слева внизу. Проводим в нём диагональ. Копируем полученную фигуру, переворачиваем её снизу вверх, и помещаем наверху. Эту же фигуру переворачиваем слева направо и помещаем внизу справа. Получили именно то, о чём РедСтар нам втолковывал, а мы не смогли дотумкать. Вот оно! И никакой векторной алгебры не надо, мечта РедСтара булет воплощена! и Ro=Rа (по углу наклона) !!! и Ra поделена строго пополам точкой РТ! Красота! Аплодисменты! Редстарыч, Нобелевку - пополам!

XMR: Круто!!!

Пермяк: Посчитал с применением "метода Плюмбума-РедСтара".

RedStar: Плюмбум пишет: я разгадал великую тайну Если бы так было просто. Ну а как, если Ra другое? Хотя подумали верно. С этого я начинал. Продолжить желаете? Пермяк пишет: Разумеется, Ri - это не линия, не гипотенуза, и не угол. Немного добавил к рисунку, не возражаете? Очевидно, что треугольник O, OP, F (Ro) вписан в треугольник O, D, E (Ra). Поэтому расчеты более корректны, чем расчеты с отстоящим отдельно треугольником А, В, С (Ri). Дело не в треугольниках, а в их векторах. Пермяк пишет: Посчитал с применением "метода Плюмбума-РедСтара". Без моего имени Никнейма. Спасибо.

Ученик: RedStar пишет: Очевидно, что треугольник O, OP, F (Ro) вписан в треугольник O, D, E (Ra). Треугольник вписан в треугольник? Поэтому расчеты более корректны, чем расчеты с отстоящим отдельно треугольником А, В, С (Ri). Для твоих расчётов Теорема Пифагора видоизменяется??? Не приписывайте мне того, чего не говорю. Тогда потрудись объяснить, каким образом у тебя получается так, что для треугольника O, OP, F расчёты якобы точней, чем для треугольника А, В, С (которых, к слову, на твоём чертёжике два).Просто не хотите понять метод Просто ты вообще ничего объяснить не можешь. Что у тебя с изложением мыслей - просто кошмар. Ты до сих пор не смог объяснить, что же ты вычисляешь в своём "МЕТОДЕ". Ты можешь собраться, и написать: я определяю (вычисляю) это, это, и это? Осилишь?

Пермяк: Жаль, что RedStar упорно не желает объяснить своих действий, попробую обратиться с наводящими вопросами. Анатолий, я немного изменил твой очередной рисунок. Для удобства. Треугольник АВС сделал побольше, для удобства рассмотрения. Вправе это сделать потому, что угол α не изменился, а значит, не изменилось и Ri, т.к. Ri=ctg(α) Ты пишешь:Очевидно, что треугольник O, Oр, F вписан в треугольник O, D, E Ну, допустим, "вписан" , но почему из этого следует, что:Поэтому расчеты более корректны, чем расчеты с отстоящим отдельно треугольником А, В, С (Ri).??? Мы, однако, не видели здесь таких расчётов, увы. И эта странная фраза:Дело не в треугольниках, а в их векторах. О каких векторах речь? У треугольника есть три стороны и три угла. Если это прямоугольный треугольник, то его стороны называют:катеты и гипотенуза. Ну, можно, конечно, составить треугольник из векторов, в электротехнике это делают часто, но у тебя-то нет на рисунках никаких векторов. И ты ещё сердишься...

Плюмбум: Вот что пришло в голову. 1. Редстар настойчиво повторяет, что Ri ему не нужно. Убираю с картинки характеристический треугольник. 2. На его рисунке между линиями, обозначенными как Ro и Ra - ПРЯМОЙ УГОЛ! "Палочки должны быть попиньдикулярны!" (с) Мне кажется, что на картинке высвечивается что-то до боли знакомое... Уж не подобие ли прямоугольных треугольников ?

RedStar: Пермяк пишет:И ты ещё сердишься...Сержусь на оскорбительные выпадки. Да и на себя, за "корявый" технический. Ученик пишет:Ты можешь собраться, и написать: я определяю (вычисляю) это, это, и это? Осилишь? Насколько смогу. Пермяк пишет:но почему из этого следует, что: Ключевая точка - Ор. O, Op, F с вершиной Ор делит треугольник ODE на O, Op, E. Катет O, Op - Ro. Катет Op, E - Ra. Известно только O, F. Добавляем точку на оси ординат "I". Получаем прямоугольник O, I, Op, F. Из него вычисляем периметр О, Ор (гипотенузу). F/I=Op (Ro) Достраиваем противоположный прямоугольник F, Op, I1, E. Так как точка E неизвестна, то неизвестна вершина E. Заладим по оси абсцисс необходимую величину, примерно = 20 (по графикам http://hiend.borda.ru/?1-22-1572769330015-00000544-000-60-0#039)Выбранная величина соответствует точке J. Прямая I=I1. FJ/I=DE (Ra) полному отрезку или ее части, стремящейся к Е (не важно). (Ro+Ra)*I=E Если уже задано (выбрано) значение Е конструктивными особенностями, то выбирая наиболее подходящую РТ будет намного проще. (F/I)+(E/I)*I=E По оси ординат (анодному току) сложнее расчет. Главное - хотя бы примерно определить конечные точки амплитуды. Об этом позже. Пока что так, как мог. Плюмбум пишет:Уж не подобие ли прямоугольных треугольников ? Нет. Ro не обязательно должно делить угол по 90 градусов отрезка DE.

r9o-11: А откуда известна точка I1, если E неизвестна? Для чего нужна I1? И что такое "20"? По ссылке это число не упоминается. Или это амплитуда напряжения? Ну и, наверное, "...вычисляем пАрАметр О, Ор (гипотенузу)...", потому как у линии не может быть периметра. Если параметр, то наверное, это длина в каких-то единицах?

RedStar: r9o-11 пишет:А откуда известна точка I1 Она - на вертикали, проведённой из точки Е. А точка Е - на пересечении линии Ra c осью абсцисс.если E неизвестна Найти Е можно определившись Ор и требуемой длины FJ (выбранной выходной амплитуды напряжения) по этой же оси абсцисс. Соответственно, катет FE - будет значением падения напряжения. И, как известно: FE/I=Raпотому как у линии не может быть периметра...Не Линия, а периметр прямоугольника.

RedStar: r9o-11 пишет: А откуда известна точка I1 Она известна по всей оси абсцисс. Так как это прямая линия I - Iбесконечность если E неизвестна Читайте внимательно. Найти Е можно определившись Ор и требуемой длины FJ (выбранной выходной амплитуды напряжения) по этой же оси абсцисс. Соответственно, катет F, E - будет значением падения напряжения. И, как известно: FE/I=Raпотому как у линии не может быть периметра... Не Линия, а периметр прямоугольника.

r9o-11: Так я, вроде читаю:RedStar пишет: Достраиваем противоположный прямоугольник F, Op, I1, E. Так как точка E неизвестна, то неизвестна вершина E. Где логика последовательности принятия решений? Как Вы всё любите запутывать... Да читая ответы ничего понятнее не становится... Ну как так-то?

RedStar: r9o-11 пишет: Где логика последовательности принятия решений? Определение периметра Op, E из прямоугольника F, Op, I1, E. Которая является Ra. Сумма двух векторов O, Op и Op, E помноженная на "ключевую" вершину Ор даст координату точки Е. (что то коряво написал...) Здесь не хватает точек по оси ординат. Если перенести данные на ось ординат с оси абсцисс, но применяя значения оси ординат, то получим результат по оси ординат. То есть величины для анодного тока, которые мы сами определяем, как и величины анодного напряжения по оси абсцисс.Как Вы всё любите запутывать...Не запутываю. Это математика... Не все знаю про вектора (еще только учусь). Думаю, что теперь принцип понятен. Можно далее приступать для обобщения с применением коэффициентами изменения тока анода и напряжения анода при изменении сеточного напряжения. Может тогда станет более понятно. Хотя и так явно видно основу методики. Так как же влияет Ri на Ra? (не надо сюда добавлять Альфу, о ней можно тоже поспорить отдельно).

r9o-11: Извините, я основу методики не увидел и принципа не понял, так как не понял по каким правилам происходит сложение и перемножение значений... Погорячился спрашивать. Зайду ещё раз через полгода-год...

RedStar: r9o-11 пишет:я основу методики не увидел и принципа не понял Это ваше право. Основа по ссылке выше на этой странице. Принцип изложил, хоть и своими словами. Это только начало анализа методики, применительно к резистивному каскадуЗайду ещё раз через полгода-год... Добро пожаловать потом.

Пермяк: RedStar пишет: Это только начало анализа методики, применительно к резистивному каскаду . Хорошо начинаете, Анатолий. Но продолжать не надо. Хватит с Вас... Завязывайте.

RedStar: Хорошо. Не раз говорил, что не поймете. А воз и ныне там. В прошлом, в болоте. П.С. Еще. Почему пентод в резистивном каскаде, не зашунтированным конденсатором катодного резистора, не приобретает свойства триода (по ВАХ)? Как в выходном каскаде с Местной ОС.

Пермяк: У пентода очень высокое Ri (десятки - до сотни кОм. Резистор в катоде образует ПОСЛЕДОВАТЕЛЬНУЮ ООС ПО ТОКУ. Такая ООС увеличивает и без того высокое внутреннее пентода. Конденсатор в катоде, устранив эту ООС, только сохраняет исходные пентодные свойства.

RedStar: Пермяк пишет: Такая ООС увеличивает и без того высокое внутреннее пентода. Тоже самое и в выходном каскаде с обмоткой КО, где ее сопротивление меняется от частоты. Пентод остается пентодом. И никакие изменения, по ВАХ в триодные, не происходит. Только не надо вводить Ri сюда. Это статический параметр лампы. Как неоднократно говорили на форуме, что РТ, по постоянке, не изменяется. С чего оно должно "плавать"? Из-за нестабильности питающего напряжения, коими тогда являлись кенотронами? Ri возможно только при повышении напряжения анода с повышением тока анода, или наоборот. Что не может соответствовать по нагрузочной Ra. Скажите, что это бред. И на этом закончим.

Пермяк: RedStar пишет: То же самое и в выходном каскаде с обмоткой КО, где ее сопротивление меняется от частоты. КО - это последовательная ООС ПО НАПРЯЖЕНИЮ, такая ООС снижвет внутренне лампы. Влияние Rакт КО здесь намного меньше (в десятки раз меньше, чем ООС с КО. RedStar пишет: Ri возможно только при повышении напряжения анода с повышением тока анода, или наоборот. Сам-то подумал, что такое написал?

RedStar: Пермяк пишет:КО - это последовательная ООС ПО НАПРЯЖЕНИЮ, такая ООС снижвет внутренне лампы. Допустим, КО имеет индуктивное к сопротивление, не активное. Они последовательно соединены Ri+Zk. Как же снижается внутреннее у выходного каскада, а у резистивного, - нет?

Пермяк: КО -часть первички, перенесённая в катод. ЧЕРЕЗ неё идёт полный анодный ток. Этот ток создаёт на катоде переменное напряжение, которое прикладывается К СЕТКЕ. ООС по напряжению понижает Ri лампы. Индуктивное сопротивление ВСЕЙ первички должно быть достаточно велико, чтобы исключить заметное снижение отдачи на НЧ. КО включена в катод. Со стороны катода лампа имеет очень небольшое входное сопротивление, которое шунтирует катодную часть первички по ПЕРЕМЕНКЕ, и индуктивность КО не играет рояли. А в резистивном каскаде, как я и написАл - ПОСЛЕДОВАТЕЛЬНАЯ ООС ПО ТОКУ. Научитесь же, наконец, отличать, в чём разница видов ООС и их действие. Трудно с Вами.

XMR: RedStar пишет: Еще. Почему пентод в резистивном каскаде, не зашунтированным конденсатором катодного резистора, не приобретает свойства триода (по ВАХ)? Как в выходном каскаде с Местной ОС. Потому что это ОС по току , он еще пентоднее становится.

XMR: Оооооо , дочитал до конца, так у Редстара полное непонимание как работает ОС и КО про которую он пишет, да какая там уж методика, плакать надо.

RedStar: Пермяк пишет: КО -часть первички, перенесённая в катод...Так. А в резистивном не течет полный анодный ток?

XMR: Редстар неужели ты не знал что резистор в катоде - это ос по току, а ко - по напряжению? я вообще как-то в шоке теперь... Вот если в катод поставить дроссель вместо резистора, то будет ос по току на индуктивности при переменном сигнале не увеличив RI при этом, а когда обмотка КО находится на одном железе с анодной и по ней течет анодный ток, то это ОС по напряжению и только по нему.

RedStar: XMR пишет: резистор это ос по току а ко по напряжению В любом случае, через КО и катодный резистор течет один ток. Разве не так? Почему лампа имеет ток покоя не связанный с Ri, а связанный с P - (мощность рассеяния анода), где как раз присутствует Ro?

XMR: RedStar пишет:В любом случае, через КО и катодный резистор течет один ток. Разве не так По переменному току части анодной и катодной обмотки включены паралельно и находясь на одном железе находятся в тесной индуктивной связи, ну то есть они включены ПАРАЛЕЛЬНО. А что бы включить ПОСЛЕДОВАТЕЛЬНО по переменному току нужно в катод как я выше сказал поставить отдельный независимый дроссель. на отдельном железе.

volli: RedStar пишет:Это же насколько надо быть уверенным в непоколебимости основ прошлого века? Появились сомнения на счёт "закона дедушки Ома"?

RedStar: XMR пишет:по переменному току части анодной и катодной обмотки включены параллельно Кто нарисует эквивалентную схему этого? Не могу понять, почему два последовательных сопротивления Ri+Zk являются параллельными? В резистивном отсутствует связь между ними. В трансформаторном - индуктивная связь, но по току такая же. По напряжению? Как считать, Анод-катод, - это одно. Анод-земля, - другое. В первом случае амплитуды складываются, во втором - вычитаются. П.С. Чувствую, не понимаете меня. Что же, пора остановиться, совсем. Будьте добры друг-другу! Спасибо всем.

volli: RedStar пишет: Допустим, КО имеет индуктивное к сопротивление, не активное.В любом случае, через КО и катодный резистор течет один ток. Разве не так?Не могу понять, почему два последовательных сопротивления Ri+Zk являются параллельными? Не может ли Анатолий пояснить, какой ток в каждом из случаев он имеет в виду? Нет ли путаницы в понятиях разности реакции различных нагрузок на переменный и постоянный ток? Не отсюда ли весь "геморой"?

XMR: Редстар ты реально глуп, без обид. глуп не тот кто не понимает а тот кто не хочет понимать.

XMR: Как ты не можешь понять что обмотки ко и анодная на одном железе, а лампа имеет RI которое ты не признаёшь и вот именно из-за этого RI обмотки включены паралельно и между ними есть индуктивная связь по типу трансформатора.

RedStar: XMR пишет:вот именно из-за этого RI обмотки включены паралЛельно Как они параллельны? Нарисуйте график, эквивалентную схему. Подробно может кто объяснить? Не надо мне говорить о выходном сопротивлении и Альфе. Все притянуто формулами. Что это за Ri, и почему она нужна? Ну не вижу связи, совсем.

Ученик: XMR пишет: Редстар ты реально глуп, без обид. глуп не тот кто не понимает а тот кто не хочет понимать. Ну вот, убедились, что я не придираюсь к редстару? Он в очередной раз не смог пояснить, что же он рассчитывает - токи, напряжения, нагрузку, Ri, какие-то зависимости, или что-то другое, применимое к электротехнике. Не-а, треугольники, прямоугольники, периметры, "вектор" треугольника... Три!!!! года РедСтар своими глупостями полощет мозги всему форуму. Сдвигов не вижу, упрям, необучаем, не понимает простых вещей, не способен изложить свои мысли. Ещё и посылает разом всех. Кто как хочет, я отправляю его в полный игнор.

XMR: Ученик, да я если честно поражён Вашему и других Коллег терпению.

RedStar: Ученик пишет:что же он рассчитывает Ученик, все вы поняли. Только вам на это наплевать, честно. Но если вам не видно что и как рассчитывается, не надо говорить за всех. Остальные молчат. Кто же попрет против всезнающих и "уважаемых гуру"? Можете сносить тему. ...с большущей колокольни... Говорил же, стоит ли продолжать? Вынудили. Самые протоптанные пути - в никуда. Счастливого пути всем!

aleks8845: RedStar пишет: И никакие изменения, по ВАХ в триодные, не происходит. Только не надо вводить Ri сюда. Я так и не смог понять, речь идет об Ri лампы как о ненужном параметре при расчетах или вообще , как не нужный параметр?

Aleph: aleks8845 пишет:Я так и не смог понять, речь идет об Ri лампы как о ненужном параметре при расчетах или вообще , как не нужный параметр? Для меня даже дико не понимать, что лампа имеет внутреннее. Это же основа. Меняя потенциал на сетке мы заставляем лампу менять своё сопротивление. А как иначе сигнал образуется? Как вообще можно быть электрическим прибором и не иметь сопротивления? Ни нулевого, ни отрицательного, никакого? Банальный пример - низкоомный триод 6с41с и первичка ТВЗ. Ток покоя 120 мА, питание 220 Вольт. Внутренне лампы, в которое RS похоже не верит, в этом режиме около 220 Ом. Почему мы для триода пытаемся сделать сопротивление обмотки как можно ниже? Подумаешь упадёт пара вольт на омическом провода, не смертельно. Вот только почему-то по переменке образуется делитель и лишь часть усиленного сигнала идёт в нагрузку. Одним резистором является пересчитанная на ТВЗ нагрузка, ну а вторым может быть только внутреннее лампы. С высоким внутренним пентода это не так явно видно, но тоже имеет место. Я предпочитаю доказательства вроде приведенных выше. Не надо практически никаких цифр и графиков. Приложили так сказать действие и смотрим что получится. В нашем случае сигнал себя повёл будто в схеме по переменке есть делитель. Смотрим какие в цепи есть сопротивления, если видим только одно, значит ещё какой-то элемент цепи ведёт себя как сопротивление. В цепи у нас лампа, нагрузка и очень низкоомный источник питания, почти КЗ. Думаю это доказывает существование внутреннего сопротивления лампы. Ну а уже зная что параметр существует можно снимать его график самому, или верить готовому.

RedStar: Aleph пишет: Как вообще можно быть электрическим прибором и не иметь сопротивления? Ни нулевого, ни отрицательного, никакого?Я отрицаю только наличие Ri.Меняя потенциал на сетке мы заставляем лампу менять своё сопротивление.Какое сопротивление? Под нагрузкой или в процессе составления ВАХ? С постоянными напряжениями сетки или с переменными? Приводили график, где видно, что такое Ri, Ra, Ro. Кто в нем видит несоответствие? Достало уже все.

XMR: RedStar пишет:отрицаю только наличие Ri Как можно отрицать Ri, оно существует независимо от твоего желания.

Пермяк: XMR пишет: По переменному току части анодной и катодной обмотки включены паралельно Ошибаетесь, сорри... Когда-то я уже выкладывал РедСтару этот рисунок: Рис.1. - Каскад с КО. Рис.2. - Точки +Ea и "земля", как известно(!) соединены ПО ПЕРЕМЕНКЕ между собой через нулевое сопротивление БП, т.е. схемка - по переменному току. Рис.3. - та же схема, показывающая, что Wa и Wк соединены между собой ПОСЛЕДОВАТЕЛЬНО-синфазно. Ну, а доказательство, какая там действует ООС - по напряжению или по току, параллельная или последовательная - это предмет отдельного разговора. Про это всё доходчиво написано в книге "Красный Цыкин", стр. 278-279 (мелким шрифтом), но это не для Редстара.

Aleph: Очень помогает собственноручно сделать таблицу комбинаций ОС. Способ ввода,вывода, по току, или напряжению. И что происходит с входным и выходным сопротивлениями, с искажениями, как выглядит ОС на схеме (резистивный делитель скажем, или КО, УЛ отвод...) Когда своими руками распишешь больше в голове остаётся. Держишь табличку при себе и благодаря ей не пишешь явных глупостей

XMR: Пермяк пишет: Ошибаетесь, сорри.. Пермяк, ну я не боюсь ошибаться, изучу вопрос, вникну, спасибо за разъяснение... Ну да, последовательно, но я имел в виду, что они на одном железе и между ними индуктивная связь имеется, или я не правильно думаю, и просто нагрузка делится так как со стороны катода внутренее лампы меньше, но получается и уменьшается как паралельное соединение двух резисторов. поправьте если гоню.

Пермяк: XMR, схемка №3 показывает, что нагрузка получает сумму напряжений с анодной части обмотки и с катодной. Это вообще единый трансформатор. Неделимый. А вот в сеточную цепь отводится, конечно, только часть анодного напряжения. Aleph пишет:Очень помогает собственноручно сделать таблицу комбинаций ОС. Способ ввода,вывода, по току, или напряжению. И что происходит с входным и выходным сопротивлениями....Такую? Ку и Кг во всех случаях снижаются, т.к. ОС - таки отрицательная. Последовательная и параллельная - это по способу подачи на вход. По напряжению и по току - по способу снятия с выхода (сигнал ОС пропорционален вых. напряженнию или пропорционален вых. току).

RedStar: Однажды было сказано в теме про "Ул-резистивный драйвер": Бокарёв Александр пишет:Редстарыч, вы делаете устройство , задача которого - звучать и радовать вас звучанием. и если его параметры случайно не укладываются в общепризнанные нормы, то у вас два выхода: или довести ваше устройство до этих общепризнанных норм и слушать, убеждая себя, что это и есть настоящий звук. Либо тихо послать эти общепризнанные нормы подальше и слушать любимую музыку. Примерно так. ваш АБ.Вот смотрю, считаю, примеряю с практикой, а вывод остается прост: "тихо послать эти общепризнанные нормы подальше". Как то так. Может скоро поделюсь новыми Практическими работами.

Пермяк: RedStar , Вы цитируете А.Бокарёва? Очень правильно. При этом хорошо бы Вам учесть, что Александр никогда не лез в Теорию, не пытался опровергать КНИГИ, и тем более - не выдумывал собственных теорий. Берите пример!.

RedStar: Пермяк пишет: Александр никогда не лез в Теорию ... не выдумывал собственных теорий Как же без Теории, методом подбора конструировать? Да кто же признается, что есть у каждого свои методы?

Фурман: RedStar пишет: Сколько сталкиваюсь, столько прихожу к мнению, что знать Ri нет необходимости. и здесь же RedStar пишет: Как же без ТеорииВ связи с этим, хочется задать такой вопрос: Имеем Усилитель SE на 6п14п-Е, с переключателем "пентод/триод" подключением экрана к аноду. Питание 250в, фикс-смещение -6,4в, ток покоя 48мА, ТВЗ 5кОм/8ом. Вопрос Редстару: почему в пентоде выходное сопротивление усилителя равно 48 ом, а в триоде 2,4 ома?

RedStar: Начну со встречного вопроса. Как проводили измерение? Опишите подробнее. На макетке могу повторить с этой же лампой.

Фурман: Измерение проводилось методом двух нагрузок, 6 и 9 Ом - в триодном включении, 6 и 12 Ом - в пентодном, на частоте 600 Герц. ТВЗ имел Ктр= 25, 5к/8ом с учётом активных сопротивлений обмоток. https://postimg.cc/Vd4JD8MR Если захотите повторить - можете применить любой ТВЗ. Питание и смещение тоже большого значения не имеют, главное - чтобы они были одинаковыми при триоде и при пентоде. Главное здесь - сравнить результаты полученных Rвых, а не их точность. Добавлю: и разумеется без всяких ООС !!!

RedStar: Эту методику хорошо знаю. Вообще то нужно дополнение. 1. Вторичная обмотка была одна для этих типов нагрузки, без отводов? 2. Какое напряжение раскачки в обоих включениях? 3. Анодное амплитудное напряжения в этих же случаях. 4. Почему в пентоде 12 Ом, а не 9, как в триодном? От этого тоже зависит. 5. Выходные напряжения под нагрузками во всех подключениях. Всё это очень важно. Иначе разговор окажется тупиковым.

Фурман: RedStar пишет: 1. Вторичная обмотка была одна для этих типов нагрузки, без отводов? Разумеется. ТВЗ - 5к/8ом, сколько же можно повторять? 2. Какое напряжение раскачки в обоих включениях? Раскачка была разной для пентода и триода, но выходное напряжение было 2 вольта, в обоих случаях. Но можно любое, важно, чтобы было одинаковое. Разумеется - не на полную раскачку.3. Анодное амплитудное напряжения в этих же случаях. Анодное не измерял, только выходное. Мы же выходное сопротивление измеряем..4. Почему в пентоде 12 Ом, а не 9, как в триодном? От этого тоже зависит. Не 12, а 6 и 12, т.е по обе стороны от номинальных 8-ми. Но и это тоже не обязательно. 5. Выходные напряжения под нагрузками во всех подключениях. А как же без нагрузки-то, милый мой? Выходное сопротивление - это величина, характеризующая реакцию выходной цепи на изменение нагрузки.

RedStar: Фурман пишет:Разумеется. ТВЗ - 5к/8ом, Значит Ктр изменяется. Далее,... При 6 Ом нагрузки должно получиться 3750. При 9 Ом - 5625... Ток лампы то больше, то меньше. Амплитуда Iвторички/Ктр=Iпервички. Само собой выходное напряжение будет изменяться.Раскачка была разной для пентода и триода, - разной!? Подробнее напишите, это интересно ведь, не правда ли почему?но выходное напряжение было 2 вольта, в обоих случаях. Как это? Вы же пишете, что вых. сопрот. значительно отличается, но сейчас 2 вольта в обоих случаях?Но можно любое, важно, чтобы было одинаковое. Конечно важно. Как же получили тогда разное сопротивление?Анодное не измерял, Вот это плохо. Так как оно будет отличаться, которое зависит от типа включения лампы. Тогда анодная нагрузка будет совсем другой. Но у вас, в трафе, неизменный Ктр, т.е. количество витков.Мы же выходное сопротивление измеряем.. Да. Только не учли мною выше приведенное.Не 12, а 6 и 12, т.е по обе стороны от номинальных 8-ми. Но и это тоже не обязательно. Как это не обязательно? А КтрВыходное сопротивление - это величина, характеризующая реакцию выходной цепи на изменение нагрузки.В таком случае, лучше подсоединить динамик. Не учли то, что нагрузка должна быть ориентирована под свое Ктр. Прошу прощения, но это не верные и не полные измерения, вами предоставленные. П.С. Простите за мой каламбур. Надеюсь, смысл понятен, хоть немного.

Фурман: На, прочитай ещё раз, и делай, как написано !!!

RedStar: Не надо НАкать. На заборе тоже написано, но не означает, что это там есть. Почему же вы не принимаете во внимание другие факторы, о которых вас спрашиваю? Скажете зачем? Еще раз прочтите, что от вас требуется для измерения, что бы состоялся правильный и адекватный разговор. П.С. Я могу извиниться, если будут доказательства правильности измерения. Но пока не вижу этого.

r9o-11: А ничего, что акустика обычно не только имеет реактивное сопротивление (т.е., разное на разных частотах), но и указанные на ней значения 2,4,6,8 или 16 Ом - это примерные значения. Тестером когда измеряете, что показывает?

Пермяк: Фурман , я тебя предупреждал: пациент безнадёжен... RedStar пишет: На заборе тоже написано, но не означает, что это там есть.Замечательный аргумент! Приравнивать надписи на заборе к статьям в технической литературе - Ваш конёк. Зачем вообще пишете на форуме, и зачем здесь задаёте вопросы? r9o-11 пишет: акустика обычно не только имеет реактивное сопротивление (т.е., разное на разных частотах), но и указанные на ней значения 2,4,6,8 или 16 Ом - это примерные значения. Мы этого не понимаем! Выдумки всё это. Написано 8 Ом - значит так оно и есть!

RedStar: r9o-11 пишет:А ничего, что акустика обычно не только имеет реактивное сопротивление Так выше указал, подсоедините дин и посмотрите. С учетом фильтра колонок, где выравнивается импеданс, нагрузка должна незначительно меняться Пермяк пишет: пациент безнадёжен...С чего это? Продолжу. Если траф рассчитан на одну нагрузку, то зачем ему "пихать" другую. И, прежде, чем отвечу, хотелось бы увидеть измерения. Вы, и практически все на форуме, толкуете, что надо придерживаться одного. Соответственно, под другую нагрузку, либо делать отводы вторички, либо перематывать траф. Сложно сделать отводы и измерить с ними под каждую им предназначенную нагрузку? Конечно, результаты не особо обрадуют, так как вых. сопр. не особо то изменится.Мы этого не понимаем! Как сказать..., а додумать сложно видимо. Не торопитесь с выводами. П.С. Подожду, что ответит Фурман. Интересно все же узнать, какие у него выходные данные при неизменных входных!!!

r9o-11: Так а какие доказательства правильности измерения требуются? И что значит эта правильность? Вот, подсоединил и посмотрел акустику на шириках - какие значения будем считать правильными, те, что на 1 кГц?: Здесь показаны сопротивления напрямую включенных динамиков 4А28, через LRC цепь (для коррекции на СЧ) и с подключенными RC цепями (для коррекции сопротивления на ВЧ).

RedStar: r9o-11 пишет: какие доказательства правильности измерения требуются?... Почему все читают вскользь? Выше постами написано же. У Фурмана сказано, что он добивался одного напряжения в 2 Вольта при разных подключениях: Раскачка была разной для пентода и триода, но выходное напряжение было 2 вольта, в обоих случаях. То есть, у него, происходит намеренное изменение во входной части, что бы достичь желаемого на выходе? Получается, что (U2-U1)=(2-2)=0!? Какие 48 Ом и 2,4 Ом? П.С. Вот потому прошу от него ответа на перечисленное мной выше. Либо он ответил с ошибкой в тексте.

r9o-11: Почему Вы считаете, что все читают вскользь? Выше же написано, что выходное напряжение было 2 вольта в обоих случаях. Какая разница, выставить на выходе 1,9 вольта или 2,1 вольта? В данном измерении важна разница выходных напряжений при изменении сопротивления нагрузки. И если выходное сопротивление изменилось, то о чём это говорит? Какие изменения произошли в лампе?

aleks8845: r9o-11 пишет: если выходное сопротивление изменилось, то о чём это говорит? Видимо RedStar понимает изменение R вых УМ, как изменение R самой нагрузки УМ (активное, реактивное) , отсюда и недоумение ,что такое R вых источника сигнала, усилителя....

r9o-11: ...Так, наверное, не бывает... Слишком альтернативно...

RedStar: r9o-11 пишет:В данном измерении важна разница выходных напряжений при изменении сопротивления нагрузки. Только эти измерения должны сопровождаться неизменными входными! Амплитудные значения тока и напряжения меняются... Все написал выше, зачем повторяться. aleks8845 пишет:изменение R вых УМ, как изменение R самой нагрузки УМ Здесь не понял. Я вот писал:Получается, что (U2-U1)=(2-2)=0!? Какие 48 Ом и 2,4 Ом? Видимо поторопился немного, извиняюсь конечно. Автор вопроса предполагал установленное выходное напряжение под Одной расчетной нагрузкой, которое изменяется под другой? Хотя не важно. Смысла отвечать нет, даже подумать над этим прав-неправ, так как нет вводных данных об усилителе. П.С. Простите, но метод двух нагрузок не учитывает большинства параметров. Только начинаю это понимать, читая Книги и форумы с обсуждениями темы. Вопрос. С чего решено получать вых. сопр. таким методом. Ищу ответ на его решение и связанные с ними мотивации.

r9o-11: Может, потому что на самом деле без разницы как определять? Не нравится метод двух нагрузок, возьмите метод одной нагрузки:

Сергеев Сергей: Может я не прав, но мне почему-то кажется что не все понимают что такое вых сопротивление усилителя. https://habr.com/ru/company/soundpal/blog/378219/ https://www.lcard.ru/lexicon/res_outp Отсюда и непонимание Ri. Для измерения вых сопротивления (сравнения) важно равенство вых напряжения. При этом у пентода и триода будет разное выходное. Из-за разного усиления.

Пермяк: Формула №3: http://hiend.borda.ru/?1-2-1575431519380-00000513-000-0-0#000.001

Плюмбум: Леонид, такие картинки - не для наших средних умов

volli: RedStar пишет:Здесь не понял. Если бы только здесь! Только эти измерения должны сопровождаться неизменными входными! RedStar пишет: Простите, но метод двух нагрузок не учитывает большинства параметров. Только начинаю это понимать, Судя по этому: RedStar пишет: С чего решено получать вых. сопр. таким методом. Ищу ответ на его решение и связанные с ними мотивации. - так ничего и не понял. Ну не укладывается это "старьё" в новую "теорию"!

RedStar: volli пишет:Если бы только здесь!А вы поняли сказанное? Ну не укладывается это "старьё" в новую "теорию"!Кто бы говорил. Плановая экономика рулит во всей красе до сих пор, запирая подальше огромное количество нововведений. Потому и наша страна в числе отсталых напрочь. Все здесь больше говорят теорией, чем делают практически. На форуме вообще перестали показывать результаты работ. Больше словоблудие ни о чем. Я не прав? П.С. Выходное больше зависит от типа лампы. Пентод или триод. Там скорее другие обстоятельства, чем Ri. Вот до них и докопаюсь со временем. Мне это ИНТЕРЕСНО! Просто, для себя. Вы же не хотите просто послушать? Только не надо оскорблений. На себя, все, посмотрите. Хотя бы немного со стороны. Много ошибаюсь в вопросах лампостроения, теоретически. Но практически многое доказывается обратное. Если не верите, так неХ... переходить на личности, т.к. у многих отсутствует практика с пересчетом теории. Что не так?

Пермяк: RedStar пишет: Простите, но метод двух нагрузок не учитывает большинства параметров. Этот метод как раз учитывает ВСЕ параметры каскада, даже те, которые не входят в формулу. Потому, что он ПРАКТИЧЕСКИЙ, т.е. его-то Вы и должны бы уважать.:) Метод двух нагрузок - не единственный. подобный ему - метод холостой ход - нагрузка. Даёт менее точный результат, но в некоторых случаях вполне может применяться. Есть и третий метод, при котором сигнал (переменный ток) подают на выход. Хорош тогда, когда ожидается очень малое значение выходного сопротивления, напр. - в транзисторных усилителях с их многочисленными глубокими ООС. Ну, и наконец, есть метод х.х.-к.з. , который так напугал Вас недавно, но Вам забыли сказать, что применяется он преимущественно "мысленно", при выводе формул, хотя в редких случаях может применять и практически.

RedStar: Пермяк пишет:Этот метод как раз учитывает ВСЕ параметры каскада, даже те, которые не входят в формулу. Допустим. Траф намотан под нагрузку 8 Ом. Ему на выход нагружаем 6 или 4 Ом. Соответственно меняются входные параметры. Верно? И смысл в методе двух нагрузок? Так почему нельзя намотать траф с отводом, и при этом измерять под свою нагрузку? Напряжение-сопротивление при 8 Ом с одним отводом и напряжение-сопротивление при 6-4 Ом со своими отводами. Тогда формула должна немного измениться.

Пермяк: Добавлю к моему предыдущему посту: https://www.hi-fi.ru/audioportal/topic/6114-vyhodnoe-soprotivlenie-lampovogo-usilitelya/page/5/ ЗЫ. Анатолий, убедительная просьба: не употребляйте при мне слово "траф.

r9o-11: RedStar, Вы же, наверное, знаете что такое пропорции и как рассчитывать резистивные делители из двух резисторов? И, наверное, когда известны U1, U2 и R1, то не составит труда вычислить неизвестное Rx? А в случае, если Rх находится внутри источника питания и Вы знаете только R1 и U2? Разве не логично будет посчитать методом двух разных нагрузок?

Пермяк: RedStar пишет: смысл в методе двух нагрузок? Что значит - "смысл"? Усилитель своим выходным сопром ДЕМПФИРУЕТ подвижную систему динамика, не позволяет ему резонировать на частоте собственного резонанса, сглаживает АЧХ на НЧ. Популярные схемы без ООС могут иметь повышенное Rвых, чтобы убедиться в том, что Кдемпф достаточен, надо уметь это Rвых измерять по факту, особенно на НЧ.

RedStar: r9o-11 пишет:Разве не логично будет посчитать методом двух разных нагрузок?Не логично. Если R1 не соответствует нагрузке, значит можно определить наиболее подходящую нагрузку. Но никак не выходное сопротивление. Лампа там не единственный элемент, верно? Rх покажет не выходное, а изменение внутренних параметров нагружаемого источника, на котором происходит то падение напряжения, которое является комплексным. Токи, напряжения, трансформатор, лампа... Комплекс должен соответствовать нагрузке. Как можно привязывать целую систему под одну только лампу? Остальные методы пока не рассматриваю, как наименее состоятельные. Пермяк пишет:Усилитель своим выходным сопром ДЕМПФИРУЕТ подвижную систему динамика,... Да не об этом. Смысл в том, что рассчитанный под определенное сопротивление усилитель, может не удовлетворять требованиям других нагрузок. Вот потому и спорю, что это не выход из положения определять методом двух нагрузок при одной рассчитанной.

r9o-11: RedStar пишет:Если R1 не соответствует нагрузке, значит можно определить наиболее подходящую нагрузку. Но никак не выходное сопротивление. Что означает словосочетание "можно определить". И что такое "подходящая нагрузка"? Для чего подходящая?

Кузьмич: r9o-11 пишет:что такое "подходящая нагрузка"? Для чего подходящая? Понимаю так: Когда делается бездумно, не проводя каких-либо расчётов, в том числе и ожидаемого выходного сопротивления, как говорится, я его слепила из того, что было... Вот тогда и встаёт вопрос, для чего подходит. А вообще, в который раз говорю, что этот тупица похоже ещё и умышленно устраивает этот базар. Он ему видимо нравится.

Плюмбум: Да он вообще задаёт вопросы только для того, чтобы тут же, не задумываясь, опровергнуть ответы. Люди для него стараются, пытаются ответить, ссылки дают, сканы выкладывают... а он даже не утруждается смысл написанного понять.

RedStar: Кузьмич пишет: этот тупица похоже ещё и умышленно устраивает этот базар. Тупица здесь вы. Не раз говорил, на личности не переходить. Когда делается бездумно, Бездумно делаете вы. Хотя... вы ничего не делаете.

r9o-11: Ну так что там с вопросами? Вы хоть скажите "можно определить" - это подразумевается "посчитать по формуле" или это в смысле "задать нужное"? Просто, в одном предложении вместе с "Если R1 не соответствует нагрузке...", вообще не понятно о чём идёт речь. Что за условие возникло "если..."?

RedStar: Плюмбум пишет: а он даже не утруждается смысл написанного понять. Не надо переворачивать... Сами попробуйте провести эксперимент с двумя нагрузками, но при отдельных отводах... ...кому говорю, все равно впустую. r9o-11 пишет:Ну так что там с вопросами? Как бы сказать? Подумаю над формулировкой предложений. П.С. Скажите, разве не прав, что:Вот потому и спорю, что это не выход из положения определять методом двух нагрузок при одной рассчитанной.

r9o-11: Так, может, ну её нафиг... Чего там думать-то? Давно бы уже взяли усь с набором резисторов через два ома и проверили расчётное Rвыходное... А то, может, мы тут сообщений настрочили больше, чем там проценты точности... А, да, ещё и резисторы, наверное, надо проверить на соответствие маркировке. А то мало ли чего на них написано...

Кузьмич: RedStar пишет:попробуйте провести эксперимент с двумя нагрузками, но при отдельных отводах... ...кому говорю, все равно впустую. Так тупице и не понятно, что от отводов меняется коэффициент трансформации. Меняется и реультат. Это коню понятно, но не тупице.

RedStar: Кузьмич пишет:...от отводов меняется коэффициент трансформации. А от разной нагрузки, что меняется? Ra. И...: Тоже - Меняется и реультат. r9o-11 пишет:Так, может, ну её нафиг...Не. Буду доводить один проект, попробую отводы сделать и проверить.

Кузьмич: RedStar пишет:А от разной нагрузки, что меняется?Меняется напряжение на нагрузке. С помощью которого и вычисляется Вых. Сопр. Ra безусловно меняется. Но в Ra входят ещё компоненты, которые лишние при вычислении Вых. Сопр. А нужны только Ктр.

RedStar: Кузьмич пишет:которые лишние при вычислении Вых. Сопр. Говорите лишние? Выходное напряжение не зависит от тока первички?

Кузьмич: RedStar пишет:Говорите лишние? Говорю. И в формуле Ra нет никаких токов, ни напряжений. Не надо усложнять себе и другим жизнь поиском блох, там где их нет. 15-страница исписана... ни о чём. Как об стенку горох.

RedStar: Кузьмич пишет:И в формуле Ra нет никаких токов, ни напряжений. Как это не зная Ктр вычислить Ra? Если только транс-тор уже готовый...

Кузьмич: RedStar пишет:Как это не зная Ктр вычислить Ra? Формулу-то смотри!!! Или ты вообще ничего не хочешь знать, видеть и замечать???

RedStar: Кузьмич пишет:Формулу-то смотри!!!Из этой формулы: Ктр=√(Ra/Rn) А как же Ra=U/I ? Там нет напряжения и тока? Вот картинка в помощь http://hiend.borda.ru/?1-2-1575480778632-00000583-000-0-0#000.001.001.001

Кузьмич: RedStar, не сваливай всё в одну кучу. Не плоди сущностей сверх необходимых.Тогда говорить не о чем.Похоже, что уже очень многие ждут, когда ты наконец замолчишь.

volli: RedStar пишет: Если R1 не соответствует нагрузке, значит можно определить наиболее подходящую нагрузку. Как? И главное - зачем? Rх покажет не выходное, а изменение внутренних параметров нагружаемого источника, на котором происходит то падение напряжения, которое является комплексным. Почему? докажите!Комплекс должен соответствовать нагрузке."С ног на голову?" А может наоборот - сначала выбрать нагрузку, а потом рассчитать соответствующий "комплекс"? А то ведь может получиться так, что к Вашему "комплексу" не окажется подходящей нагрузки? Может считаете так: 2-5Ri=RaКтр=√(Ra/Rn) Ra=U/I Что Вы имеете в виду под Ra? Не тут ли "собака" зарыта? Подсказка: Что такое "приведённое" сопротивление? и какое отношение оно имеет к Ri?

Кузьмич: volli пишет: к Вашему "комплексу" не окажется подходящей нагрузки? Вот у него как раз и оказывается. А потом при этом заявляет, что теория - фуфло.

volli: Короче, случай "клинический"! Плохо владея теорией, методикой расчёта, и техникой измерений, не соответствие расчёта и полученных результатов "измерений" своего "комплекса" решил упразднить "устаревшую классическую теорию", и создать свою новую "теорию", подогнав её под свои "результаты измерений"... . Причём чужими "мозгами"!

RedStar: volli пишет:Как? И главное - зачем?Затем, что большинство, мотая тран-торы, не всегда "попадают" в расчетную нагрузку. Иногда и я не исключение. Приходится перематывать. Но мне это не сложно. Меняя нагрузку можно определить наиболее подходящую, и потом, отматывая-доматывая вторичку выбрать оптимальный вариант.Почему? докажите! Попробуйте со вторички снять бОльший ток, если первичка не сможет ее "передать". Будет значительное понижение напряжения. Известную формулу описал выше."С ног на голову?"Вообще-то все правильно. Комплекс должен соответствовать нагрузке, а нагрузка комплексу. Да и не такой уж большой выбор нагрузок. 4, 6, 8 и редко выше. В последнем вопросе, - все в куче. Поконкретнее.Короче, случай Теория, методика расчета. Да все мне известно. И так, и так - экспериментирую. Упразднять? Это вы решили за меня. Я не предлагаю подобного, а лишь пробую дополнять и показываю Практические результаты! Создаю? Вполне. Подгонять под результаты? Как бы - да. Они точнее. А техника измерений едина, смотря как считать потом. Кузьмич пишет:теория - фуфло. За всю ни слова не сказал. Опять передергиваете.

Кузьмич: RedStar пишет:что большинство, мотая тран-торы, не всегда "попадают" в расчетную нагрузку. Не надо про всех обобщать. Расчёт и производится для того, чтобы "попасть" в расчётную нагрузку. А если ты не умеешь рассчитывать, это твоё горе.Меняя нагрузку можно определить наиболее подходящую, и потом, отматывая-доматывая вторичку выбрать оптимальный вариант.Это из серии: Дурная голова рукам и ногам покоя не даёт.если первичка не сможет ее "передать"Чушь!За всю ни слова не сказал Сказал. И много.

volli: RedStar пишет:Попробуйте со вторички снять бОльший ток, если первичка не сможет ее "передать".Объясните пожалуйста, как первичка передаёт "ток" вторичке, если они гальванически не связаны? Ещё раз:Из этой формулы: Ктр=√(Ra/Rn) А как же Ra=U/I ? Там нет напряжения и тока? Ra - что это такое, откуда Вы его взяли (получили)? Подсказки: (по системе ЕГ - выбери правильный ответ ) 1. сопротивление по постоянному току, 2. сопротивление по переменному току. 3. приведенное сопротивление. RedStar пишет:А техника измерений едина, Но Вы ею не владеете в должной степени, так как пишете:смотря как считать потом. при "подсчёте" постоянно "мешаете в кучу" (заменяя ( подгоняя) результат под нужный Вам ответ путём подмены действующего напряжения амплитудным, сопротивление по пстоянному току с сопротивлением по переменному и т.д. в том же духечто большинство, мотая тран-торы, не всегда "попадают" в расчетную нагрузку. С какой точностью ( в процентах)? Объясните, для чего такая точность с точки зрения "комплексной нагрузки" и частотной зависимости.Теория, методика расчета. Да все мне известно.Есть большие сомнения.

volli: RedStar пишет:Попробуйте со вторички снять бОльший ток, если первичка не сможет ее "передать".Объясните пожалуйста, как первичка передаёт "ток" вторичке, если они гальванически не связаны? Ещё раз: RedStar пишет:Из этой формулы: Ктр=√(Ra/Rn) А как же Ra=U/I ? Там нет напряжения и тока? Ra - что это такое, откуда Вы его взяли (получили)? Подсказки: (по системе ЕГ - выбери правильный ответ ) 1. сопротивление по постоянному току, 2. сопротивление по переменному току. 3. приведенное сопротивление.А техника измерений едина, Но Вы ею не владеете в должной степени, так как RedStar пишет: смотря как считать потом. при "подсчёте" постоянно "мешаете в кучу" (заменяя ( подгоняя) результат под нужный Вам ответ путём подмены действующего напряжения амплитудным, сопротивление по пстоанному току с сопротивлением по переменному и т.д. в том же духечто большинство, мотая тран-торы, не всегда "попадают" в расчетную нагрузку. С какой точностью ( в процентах)? Объясните, для чего такая точность с точки зрения "комплексной нагрузки" и частотной зависимости.Теория, методика расчета. Да все мне известно. Есть большие сомнения.

r9o-11: RedStar, а Вы читали опусы Владимира Пронина про его "путь в болоте флейма"?

RedStar: Кузьмич, какой же вы упертый.Не надо про всех обобщать... Так докажите делом, приведите пример. Предварительно нагрузив на 4, 6, 8 Ом и составьте таблицу. По ней видно будет, попали в яблочко или в небо. Только желательно пару-тройку тран-ров, а то выберете самый-самый. Чушь!Ну, если так считаете, значит не знаете: https://ru.wikipedia.org/wiki/%D0%9A%D0%BE%D1%8D%D1%84%D1%84%D0%B8%D1%86%D0%B8%D0%B5%D0%BD%D1%82_%D1%82%D1%80%D0%B0%D0%BD%D1%81%D1%84%D0%BE%D1%80%D0%BC%D0%B0%D1%86%D0%B8%D0%B8 Вы уверены, что под определенную нагрузку не стоит учитывать токи? volli пишет:Объясните пожалуйста, как первичка передаёт "ток" вторичке, если они гальванически не связаны? И для вас ссылка выше. Ответ.3. приведенное сопротивление. Ничего не мешаю в кучу. Считаю только амплитудными. А с точностью все прекрасно получается. r9o-11 пишет: "путь в болоте флейма" Скиньте, почитаю. П.С. Уже не надо, нашел. Цитата: "Пройти же в топях, не зная троп, не сможет никто." Так вот, не знал бы основ, не лез бы напролом.

Кузьмич: RedStar пишет:Ну, если так считаете, значит не знаете:Всё больше и больше удивляюсь, какой ты тупой и ещё глупый... Так вот попытаюсь донести в очередной раз. Если твоя первичка не способна что-то передать... то КПД твоего транса ниже плинтуса. Т.е. этот ТВЗ по КПД не рассчитывался. И актсопры этого очень высокие. Это результат твоего стремления сэкономить на нём и сделать его как можно меньше. Выручит только досканальное изучение классиков Цыкина, Войшвилло. И неукоснительное соблюдение их требований.Так докажите делом, приведите примерМожет тебе ещё и ... вареньем намазать??? В теме "ТВЗ для 6С4С" привёл пример ТВЗ. Не до конца посчитал, но это мелочи. Мотай, будет отличный ТВЗ! Сомневаешься? Это от незнания.

RedStar: Кузьмич пишет:И неукоснительное соблюдение их требований.Так соблюдайте себе на здоровье, а ко мне не лезьте. Все равно не понимаете о чем я.Мотай, будет отличный ТВЗ!Нет уж, спасибо. Получить максимум 3,3 Вт? Я бы на 4,5 сделал.

Кузьмич: RedStar пишет:Так соблюдайте себе на здоровье, а ко мне не лезьте. Нефиг тогда на техническом форуме свою бредятину нести!Нет уж, спасибо. Получить максимум 3,3 Вт? Я бы на 4,5 сделал. Ещё раз тупишь. Даже прочитать нормально не можешь. В теме все выкладки. 5,42 ватта. При КПД 95% всё больше 5 Ватт на выходе. Только чуток вторичку "подправить". Видишь какой ты тупой??? Прочитать даже не можешь.

RedStar: Кузьмич, лучше бы там расписал подробнее, как получил 5 Вт. Какие напряжения.... Скажешь, что нет необходимости, опять откажешься? r9o-11 пишет:не про те топи и тропы Вы прочитали. "А ещё надо уметь это всё воплотить". Чего не видно от окружающих.

Кузьмич: RedStar пишет:Лучше бы там расписал подробнее, как получил 5 Вт Ну ты и пенёк, чессслово... Удивлению нет предела. Там всё расписано и указано. RedStar пишет: Скажешь, что нет необходимости, опять откажешься? Тебе очень много здесь было сказано. И ещё сказали бы, если бы ты хоть немного слушал и вникал. И не придумывал свою ахинею, типа этой RedStar пишет:а ко мне не лезьте.Чего не видно от окружающих. А тебе надо чтобы к тебе на квартиру привезли, включили и заставили слушать??? А ты всё обсерешь, и скажешь, что это всё фигня, вот у тебя 6Ж2П это вещь. Да?

r9o-11: RedStar, так вопрос в том, что Вы собрались воплощать... Надо ли оно кому-то ещё кроме Вас? И это "надо" - оно точно "надо", а то может, это временная прихоть? А надо совсем другое?.. Вот примерно об этом можно почитать у Влада Пронина...

RedStar: Кузьмич, вот привели найденную по формуле мощность. А в реальности, при Ктр 29? Не нужно ко мне привозить. Покажите здесь ваши измерения на действующей модели. Тогда разговор примет другие обороты. Я же привожу измерения.

volli: RedStar пишет:Вы уверены, что под определенную нагрузку не стоит учитывать токи? Уверен. Потому как нагрузка у Вас НЕопределённая, она "гуляет" довольно в больших интервалах по разным причинам. Ваша ссылка не совсем корректна - силовые трансформаторы довольно сильно отличаются от трансформаторов "звукового" диапазона, и "тупо" применять методику расчёта силового к "звуковому", как Вы делаете, по крайней мере - невежество. Попробуйте со вторички снять бОльший ток, если первичка не сможет ее "передать". Будет значительное понижение напряжения. Известную формулу описал выше. А как насчёт соответствия ЭДС? Сама формула коэф. трансвормации через U или I имеет погрешность (Если пренебречь потерями в обмотках), достаточно приличную, а Вы пытаетесь "ловить блох" в расчёте трансформатора. RedStar пишет:Ответ. цитата: 3. приведенное сопротивление. Поясните пожалуйста: Какого и к чему? С каким коэффициентом? пишет: Из этой формулы: 1. Ктр=√(Ra/Rn) А как же 2.Ra=U/I ? Там нет напряжения и тока? А так же: Какое Ra в первой формуле, а какое во второй? Из трёх ответов по ЕГ. Можно ли их отождествлять?

Кузьмич: RedStar пишет:вот привели найденную по формуле мощность. А в реальности, при Ктр 29? Ты вообще читать умеешь? А запоминать что прочитал? И сортировать, что к чему??? Ктр = 29 для Ra=7 кОм. Мощность на скрине из учебника 5,42 Ватта при Ra = 6,24 кОм. Вторичку для этого случая надо подкорректировать, т.е. количество витков вторички несколько увеличить (до 99 витков). А проводок выбрать потоньше, что бы уместить на каркасе. При КПД ТВЗ 95% реально на выходе будет мах. мощность 5,1 ватта. Сомневаешься??? Значит опровергаешь теорию. А твоя практика, если она не соответствует теории - туфта. RedStar пишет: Тогда разговор примет другие обороты. С кем, с тобой? До тех пор, пока ты не изучишь теорию... До тех пор, пока у тебя практика не будет сходится с теорией... До тех пор, пока ты не научишься выражаться технически грамотно и без дебильного диалекта... До тех пор, пока ты не перестанешь выкладывать сюда всякую ересь.... Разговора не жди. А вот бить тебя будут. Я не исключение.

RedStar: volli пишет:силовые трансформаторы довольно сильно отличаются от трансформаторов "звукового" диапазона, Отличаются, верно подметили. Только никто не отменял Закон Фарадея и уравнения идеального трансформатора. Считаете это невежеством? Остальное тоже верно, но погрешности считаемы. Но кто их толком рассматривает? Остальное оставлю без ответа, сами знаете, что да как.

Кузьмич: RedStar пишет:Только никто не отменял Закон ФарадеяЭто что??? Попытка сделать умное лицо при отвратительной игре??? Ты простое, азы освой, изучи! Фарадей, блин!

RedStar: Кузьмич пишет: сделать умное лицо при отвратительной игре Играешь ты, словесно, без доказательств.При КПД ТВЗ 95% реально на выходе будет мах. мощность 5,1 ватта.Будет ли? Собирайте, проверим. По расчету выходит красиво, а на деле?

Кузьмич: RedStar пишет:без доказательств. Я тебе учебник привёл. Это основа! Самое главное доказательство! По расчету выходит красиво, а на деле?На деле только твои кривые руки и бестолковая голова могут всё изменить. Это ещё раз подтверждает, что ты отвергаешь теорию. Значит - не иначе, как тупица. Обратное можешь доказать?

RedStar: Кузьмич пишет:...учебник привёл. Это основа! Хорошо, допустим ты произвел расчет выходного тран-ра по теории. Есть уверенность в том, что твой расчет совпадет с реальным изделием? Ты привел формулы, в: http://hiend.borda.ru/?1-22-1575554626759-00000544-000-280-0#131.001.001.001.001.001.001 Практических доказательств правильности не приводишь. Это минус. По теории, как бы, плюс. Зачетно! Сам грешу таким, во Имя Справедливости! Только объясни, каким образом, по формуле 8.23, выходит мой Гуттаперчевый на мощность от 20 Вт? Хотя там только 10 Вт? Промолчим, или будем ссылаться на оскорбления, теорию, и отмазки твои кривые руки и бестолковая голова могут всё изменить.Кривые руки у того, кто не задумывается над процессами внутри. Дальше не буду говорить. Сам поймешь. Бестолковая голова у тех, кто беЗтолково твердит заученное. Не пытаясь думать своими проветренными мозгами. "Бес - приставка. В Русском языке означает "беса" (с маленькой буквы). А Слово Без - Отрицание". Раньше так и было, пока не внес Луначарский, и ему подобные товарищи, эту хрень.Обратное можешь доказать? Что мне доказывать? Вам мало может практики, в которых не понимаете, почему очень многое идет вразрез? В который раз сообщаю, я НЕ доказываю! Только Показываю на возможность изменения некоторых "ваших" постулатов. Не понятно в очередной раз? Форумчане! Извините меня. За настойчивость и корявый ломанный язык в терминологии. За предъявление своего мировоззрения. За пробу внедрения некоих формулировок, отличных от теории. Ошибаюсь часто. Но эти ошибки явно показывают отличия. Кто не верит или упертый, - ваше право. Ничего против не имею. Будьте благоразумны в выражениях. Спасибо. Мне все Интересно, хоть и устарело это для нашего времени. Учителя срываются. Только форум не средство обучения. И это очень жаль.

r9o-11: А, может, некоторая проблема в том, что у RedStar-а осциллограф неправильно THD показывает и он думает, что «ух ты, как у меня всё здорово с точными формулами и с катодными обмотками получается, а эти «лопухи» собирают усилители с процентными искажениями и до сих пор не слышали настоящего звука»? Ну, вот, например, берём скрин экрана измерения искажений из темы Усилитель "Kainsar-444" - 6П44С, 6Ж4 , где про синий спектр прибор говорит, что там THD 0,2819%. Но читаем синюю строку с самого начала и видим, что основной тон 1,0578 Гц имеет уровень +15,5 dBV, а его 2-я гармоника имеет уровень -20,47 dBV, т.е. разница между ними 35,97 dBV, что составляет около 63 раз. А что такое 63-я часть от 100% – правильно, 1,587%. Попробуем перепроверить «через вольты» - переведём значения dBV в условные напряжения и посчитаем их процентное отношение. Тогда пусть напряжение основного тона будет 1 В. Уровень его 2-й гармоники будет 1/63=0,01587 В, что опять же составляет 1,587% от 100% сигнала в 1 В. А что же тогда за значение 0,2819% показывает осциллограф? Смотрим синюю строку и видим, что THD там указан ещё раз, но уже в децибелах -50,9984 dB. Что это такое, к чему привязано? А если посчитать этот уровень относительно 1 В, то что получится? Округляем 50,9984 до 51 (можно же? ) и посчитав, получаем 0,28% - то самое последнее показание в строке - THD 0,2819%. Теперь вопрос – а почему осциллограф проводит расчёт THD от нулевой отметки шкалы децибел? Может, у него установки такие – привязка расчёта не к уровню основного сигнала, а к отметке «0» шкалы dBV? Ну, и ещё один вопрос - а что это за значение уровня -50,9984 dB, к чему оно относится? Где "это" на спектре? Куда смотреть? А вот это фиг его знает - наверное, надо читать инструкцию, но как-то лень…

RedStar: r9o-11 пишет: осциллограф неправильно THD показывает?А вы спросите у других людей, которые пользуются таким же прибором. Что они вам скажут?почему осциллограф проводит расчёт THD от нулевой отметки шкалы децибел? От чего же еще? Есть тема про этот осцилл., там почитайте и присоединяйтесь для обсуждениянадо читать инструкцию, но как-то лень… Вот и в инструкции сказано, что все измерения производятся на основе Общепринятых. Даже сказано, что он соответствует стандартам. Приобретите его, он не дорогой. Сами проверите. Я буду только рад, если вы измените это положение. У меня, и других, нет оснований ему не доверять.

volli: R-S пишет:Только никто не отменял Закон Фарадея и уравнения идеального трансформатора. Интересно, кто кроме Вас применяет "идеальные силовые трансформаторы" в аудио? И чего это люди "изгаляются" с аудиотрансформаторами? Кстати, Вы тоже. RS пишет:Траф намотан под нагрузку 8 Ом. Ему на выход нагружаем 6 или 4 Ом. Соответственно меняются входные параметры. Верно? Верно так же и то, что на частоте 50Гц динамик может иметь сопротивление 5 Ом, на частоте 500Гц - 10 Ом, на частоте 1000Гц - 18 Ом и т.д. Хотя на постоянном токе - 7.2 Ом-а! Вы на какой частоте предпочитаете слушать свой "комплекс"? Вот под такое частотное сопротивление и рассчитывайте трfнсформатор. Ах да, я забыл - Вы слушеете " идеальный силовой" трансформатор - тогда да, частота сети довольно стабильна, так что считайте на 50Гц и "будет Вам счастье"!

r9o-11: RedStar, так он у меня есть, но в аудиоизмерениях всё равно предпочитаю пользоваться Спектралабом. Вот смотрите, как раз Спектралабовский скрин, сделанный при проверке какого-то макета: Что мы здесь видим? Разница между основным тоном и 2-й гармоникой чуть менее 40 dB и показания THD более 1%. Вроде как, здесь теория с практикой сходятся...

RedStar: volli пишет:кто кроме Вас применяет "идеальные силовые трансформатора" в аудио? В который раз. Кто вам сказал, что применяю расчеты от силовиков? Закон трансформации НИКТО НЕ ОТМЕНЯЛ! Как малые дети...Верно так же и то, что на частоте 50Гц динамик может иметь сопротивление 5 Ом, на частоте 500Гц - 10 Ом, на частоте 1000Гц - 18 Ом и т.д Верно. Но измерения проводите на Постоянном резисторе? Комплекс почему то многие не учитывают? Увеличение нагрузки, более расчетной, к вашему примеру, только благотворнее сказывается на ток первички не принуждая лампу уменьшать ЭДС тран-ра. Есть попытка определения минимальной нагрузки, что бы обеспечить подъем индуктивного сопротивления динамика. Но промолчу, и вам не советую пока это затрагивать.Вы слушеете " идеальный силовой" трансформатор - тогда да, частота сети довольно стабильна, так что считайте на 50Гц и "будет Вам счастье"! Ошибаетесь. Рассчитываю на 30 Гц - получаю 30. Рассчитываю на 20 - получаю 20. r9o-11 пишет: мотрите, как раз Спектралабовский скрин, сделанный при проверке какого-то макет Сделайте сравнение обоих программ, без изменения входных параметров. Покажите скрины. Готов обсудить с вами (не здесь, а в теме про осцилл.) Возможно у вас не настроен прибор или проблемки в звуковой карте. Всякое бывает.разница между основным тоном и 2-й гармоникой чуть менее 40 dB и показания THD более 1%. Какой метод измерения предустановлен? Их там более 20.

r9o-11: А, кажется, я понял, почему китайские проценты такие - они вычисляют именно коэффициент нелинейных искажений сигнала, но не в процентном отношении, т.е. не умножают полученный результат на 100% ( [здесь можно почитать - там есть строка: "Коэффициент безразмерный, но обычно умножается на 100% для получения значения в %."[/url] ). А почему к этому значению они подставляют значок "%" - да фиг его знает, им, наверное, наши арабские цифры ничего не говорят - у них же своё написание чисел... Переизмерять и перепроверять я ничего не буду - пользуюсь спектралабом более 10 лет, всё проверено неоднократно на куче карт, компьютеров и измерений.

RedStar: r9o-11, Не верно, совсем. В чистом пентоде получаю реальные значения искажений до 10%. Скаждете, что там не 10%, а 100%. Даже 2% мне коробит уши так, что словно мимо меня едет Белаз! а также во многих других случаях при неправильно выбранных режимах. В триоде добивался 0,8% чистых. Предлагаю вам перенести это в тему об осцилле.Переизмерять и перепроверять я ничего не буду - пользуюсь спектралабом более 10 лет, всё проверено неоднократно. Зря. Спектралаб зависит от карты и настройки, более тщательной, чем готовый прибор.

r9o-11: Не надо ничего переносить - всё здесь написанное относится именно к тому, как все мы бываем уверенны в правильности своих измерений.

RedStar: r9o-11 пишет:все мы бываем уверенны в правильности своих измерений. Относительно. И заблуждение. А проверять чем то другим? Привыкли видимо.

r9o-11: Какой Вы нудный... Ну, ладно, минутное дело - запустил запись 1кГц сигнала с THD 1% с тестового диска и посмотрел :

RedStar: r9o-11 пишет:Какой Вы нудный...Не нудный, а дотошный. Все нравится, а многое не нравится. Нравится экспериментировать и получать удовольствие. Просто для себя. Для получения адекватных значений при применении формул. Кому как, а мне ответственно подходить надо. Не нравится - это тупое следование основам пятилеток....и посмотрел Смешного там нет ничего. Это так, о смайлах. Судя по графику, очень большой шум от БП. Хотя очень хорошо давит основную гармонику питающей сети. Вам респект! П.С. Еще раз повторю. Переносим это обсуждение в тему про осцилл. Хорошо?

r9o-11: Зачем переносить? Если не по теме, то, наверное, удалить надо. То же измерение на другой карте :

Кузьмич: RedStar пишет: Хорошо, допустим ты произвел расчет выходного тран-ра по теории. Есть уверенность в том, что твой расчет совпадет с реальным изделием?Абсолютно. У меня и получается всегда с погрешностью плюс-минус 5% в домашних условиях. И это отличный результат. Здесь на форуме есть мои посты по ТВЗ, даже индуктивность рассеяния совпала с расчётной менее чем в 5%. Со 100% -ой вероятностью невозможно изготовить даже на производстве. Этому существует ряд причин. Да это и не нужно.объясни, каким образом, по формуле 8.23, выходит мой Гуттаперчевый на мощность от 20 Вт? Хотя там только 10 Вт? А я что, экстрасенс? Или ты хочешь, что бы всю твою тему перепроверил? Ищи свои ошибки. Кто ищет, тот найдёт. И вообще, не прыгай с пята на десята, разберись с одним сначала.

RedStar: r9o-11 пишет: Если не по теме, то, наверное, удалить надо. Перенести. Так лучше было бы. В вашем графике теперь больше ВЧ шума стало. Есть общий подъем в области высших гармоник. Больше леса получили. Мне не нравится. Ужасный звук будет. Я прекращу говорить на эту тему здесь. Прошу перейти на другую. Кузьмич пишет:У меня и получается всегда с погрешностью плюс-минус 5% в домашних условиях. И это отличный результат. Может быть да, но этого не отражается в твоих разработках, коих нет на форуме. Бабка надвое сказала... Сказать? что-то и я могу, апеллируя образностью, а может своеобразным мышлением?Со 100% -ой вероятностью невозможно изготовить даже на производстве. Абсолютно согласен с тобой. Этого вряд ли кто добьется. И мне не нужны заоблачные дали.А я что, экстрасенс? Дело не в экстрасенсорике, а предоставленных, мною в теме, Измеренных значениях. Таким расчетом, о котором толкую, не владеете, что бы реально подсчитатьИли ты хочешь, что бы всю твою тему перепроверил?Зачем? Есть схема, есть данные. Сложно прочитать? Подумав хорошо, там очень просто решается.Ищи свои ошибки. Да, есть там, немного, ошибок. Признаю то, что допустил отклонение в расчетах. Не заметишь их со своим эгоцентризмом. Но это не помешает понять идею применения дополнительных обмоток, если не много уклонитесь от основ. Все просто, до банальности.

Плюмбум: RedStar пишет: Только объясни, каким образом, по формуле 8.23, выходит мой Гуттаперчевый на мощность от 20 Вт? Хотя там только 10 Вт? Считать не умеешь, вот и вся причина.

RedStar: Плюмбум пишет:Считать не умеешь, вот и вся причина.Сказали раз, скажите два. Подсчитайте, если не согласны.. Или только слова одни есть, без подкрепления?

Плюмбум: RedStar пишет: Подсчитайте, если не согласны.. Или только слова одни есть, без подкрепления? Нет, дорогОй! Ты сделал заявление на ошибочность формулы Войшвилло, ты и должен был бы показать, как считал. Но ты сделал голословное утверждение, а с меня требуешь расчёт... Далее:Из этой формулы: Ктр=√(Ra/Rn) А как же Ra=U/I ? Там нет напряжения и тока?Это формула Коэффициента трансформации. А он может быть вычислен как: - отношение напряжений Ктр=U1/U2, - отношение токов Ктр= I2/I1 - как кв. корень из отношения сопротивлений Ктр=√(Ra/Rn) В разных конкретных случаях можно пользовать любое из этих соотношений. При расчёте трансформатора удобно применить третью формулу. Тебе, правда, это сложно понять, раз такой вопрос задал...

Кузьмич: RedStar пишет:Сказать? что-то и я могу, апеллируя образностью, а может своеобразным мышлением? Чудак ты Толя, на букву МЭ. Мелешь языком всякую чушь. Кичишься китайской дешёвкой, которой правильно пользоваться не научился.Таким расчетом, о котором толкую, не владеете, что бы реально подсчитатьЭтим расчётом "владеешь" только ты. Только гордиться им не получится, сплошная лажа.Есть схема, есть данные. Сложно прочитать? Подумав хорошо, там очень просто решается.Я для тебя должен посчитать? Не много чести? Просто? А что же у тебя там ошибки???если не много уклонитесь от основ. Какая чушь???!!! Для сравнения, ты предлагаешь умножать по какой-то другой таблице умножения. Самому - не смешно? А, да... Закон Ома у тебя другой, свой... Все просто, до банальности. Ага, а то тут никто не видит. Плюмбум пишет: Это формула Коэффициента трансформации. А он может быть вычислен как: Есть ещё вариант: Ктр = W1 / W2, или n = W2 / W1 Есть ещё варианты.



полная версия страницы